Download as pdf or txt
Download as pdf or txt
You are on page 1of 83

TẬP SAN

TOÁN HỌC

Số 07
Công thức nội suy Lagrange và một số áp dụng
Đẳng giác - Liên hợp
Thuật toán tham lam
Một số đề thi và bài tập

Nguyễn Tăng Vũ - Lê Phúc Lữ - Nguyễn Tiến Hoàng


Mục lục

1 Công thức nội suy Lagrange


và một số áp dụng 3

2 Đẳng giác - Liên hợp 9

Tài liệu tham khảo 34

3 Tuyển tập số học thi HSG cấp tỉnh năm 2021 - 2022 35

4 Thuật toán tham lam 47

5 Đề thi chọn đội tuyển toán


Trường PTNK năm học 2021 51

6 Đề thi chọn đội dự tuyển trường PTNK năm 2022 57

7 Đề kiểm tra lớp 10 Chuyên đề 59

8 Một số bài tập rèn luyện 64

9 Hình học xạ ảnh


bắt nguồn từ hội họa 76

10 Giới thiệu các lớp chuyên đề 81

1
2

NGUYỄN TĂNG VŨ - LÊ PHÚC LỮ - NGUYỄN TIẾN HOÀNG

TẬP SAN TOÁN HỌC


STAR EDUCATION
Số thứ 07 - 2022

Đến hẹn lại lên, đến tháng 1 hàng năm thì các học sinh trung học phổ thông chuẩn bị
cho kì thi Học sinh giỏi Quốc gia, năm nay lịch thi lùi vào tháng 03 do tình hình dịch
bệnh vẫn còn hoành hành. Trong thời gian qua, mặc dù học online nhiều căng thẳng
nhưng nhìn chung tình hình các em học vẫn rất tốt, vẫn có sự kết nối tốt giữa thầy cô
và học sinh, giữa các học sinh với bạn bè hoặc các đàn anh đi trước. Các em trải qua
những trải nghiệm thú vị khi học online, cũng là cơ hội để các em các tỉnh khác có
thể tiếp cận các bài giảng ở các thầy cô tỉnh khác.
Trong không khí học tập sôi nổi đó, các giáo viên trợ giảng của Star Education tiếp tục
cố gắng thực hiện tiếp tục số 07 Tập san Star Education, xem như thêm chút hương
vị cho các món ăn mà các em dùng hàng ngày, đối tượng chủ yếu hướng đến là các
em học sinh chuyên toán THPT và các bạn đam mê môn toán.
Trong số này cũng tiếp tục là các chuyên mục quen thuộc: chuyên mục bài viết với
các bài viết của bạn giáo viên đang giảng dạy tại Star Education và các bài viết khách
mời, chuyên mục đề thi có Đề thi và lời giải chọn đội tuyển trường Phổ thông Năng
khiếu và một số đề thi của các tỉnh để các bạn tham khảo, Đề kiểm tra lớp 10 Chuyên
đề tại Star Edu,...Chuyên mục bài tập rèn luyện.
Ngoài bài viết của các giáo viên của Star Education, rất mong nhận được thêm sự
đóng góp của bạn đọc gần xa cho tập san để ngày một phong phú hơn, phục vụ tốt
hơn cho cộng đồng dạy và học Toán.
Dự kiến số tiếp theo, Tập san 08, sẽ được xuất bản vào tháng 05 tới với nội dung chủ
yếu dành cho các bạn chuẩn bị thi tuyển sinh 10 và tuyển sinh ĐH, xin đón nhận các
bài viết gửi về cho Ban biên tập. Mọi đóng góp xin gửi về các địa chỉ nguyentangvu@
gmail.com hoặc lephuclu@gmail.com.
Bản quyền thuộc trung tâm STAR EDUCATION, được đăng tải miễn phí trên mạng.
Mong rằng tài liệu này sẽ được đón nhận và được chia sẻ rộng rãi. Xin chân thành
cảm ơn.

Tập san Toán học STAR EDUCATION


Công thức nội suy Lagrange
và một số áp dụng

Trần Hà Sơn

Trong khuôn khổ bài viết này, chúng tôi trình bày công thức nội suy Lagrange và một số
ứng dụng của nó trong việc giải một số bài toán sơ cấp. Bài viết gồm ba phần, phần một
nêu cách xây dựng công thức nội suy Lagrange và ý nghĩa hình học, phần hai trình bày
một số ví dụ áp dụng, phần ba là một số bài tập luyện tập để bạn đọc có thể rèn luyện
thêm.

1. Công thức nội suy Lagrange


Cho x 1 , x 2 , . . . , x n , c1 , c2 , . . . , cn là các phần tử của trường K(K là trường số thực R hoặc
trường số phức C), trong đó x i 6= x j với mọi i 6= j. Tìm tất cả các đa thức f (x) ∈ K[x]
thỏa mãn f (x i ) = ci .
Đặt
Y x − xj
ψi = , 1 ≤ i ≤ n,
i6= j
x i − x j

n
X
f0 (x) = c1 ψ1 (x) + c2 ψ2 (x) + . . . + cn ψn (x) = ci ψi (x).
i=1

Khi đó đa thức f0 (x) được gọi là đa thức nội suy Lagrange hoặc công thức nội suy
Lagrange. Các số x 1 , x 2 , . . . , x n được gọi là các nút nội suy.
Ta có kết quả sau:

Định lý 1. Với giả thiết và kí hiệu trên, đa thức f (x) ∈ K[x] thỏa mãn điều kiện
f (x i ) = ci , i = 1, 2, . . . , n khi và chỉ khi f (x) có dạng

f (x) = f0 (x) + g(x)ϕ(x)

trong đó ϕ(x) = (x − x 1 )(x − x 2 ) . . . (x − x n ), g(x) là đa thức nào đó của K[x].

Chứng minh. Giả sử f (x) = f0 (x) + g(x)ϕ(x) với g(x) ∈ K[x]. Ta có ψi (x j ) = 1


nếu i = j và ψi = 0 nếu i 6= j với 0 ≤ i, j ≤ n. Do đó f0 (x i ) = ci , ϕ(x i ) = 0 với
∀i = 1, 2, 3, . . . , n.

Ngược lại, giả sử f (x) ∈ K[x] thỏa mãn điều kiện

f (x i ) = ci , ∀i = 1, 2, . . . , n.

3
4

Đặt
h(x) = f (x) − f0 (x).
Ta có f0 (x i ) = ci , do đó hi (x i ) = 0, ∀i = 1, 2, . . . , n. Do đó theo định lý Bezout ta
suy ra h(x) chia hết cho x − x i (1 ≤ i ≤ n). Suy ra h(x) chia hết cho ϕ(x), nghĩa
là tồn tại g(x) ∈ K[x] sao cho h(x) = ϕ(x)g(x). Do vậy f (x) = f0 (x) + h(x) =
f0 (x) + g(x)ϕ(x).
Từ định lý 1, nếu g(x) là đa thức không thì ta có f (x) = f0 (x) là đa thức duy nhất
thỏa mãn các điều kiện f (x i ) = ci và deg f (x) ≤ n − 1.

Ý nghĩa hình học của công thức nội suy Lagrange


Về mặt hình học, việc xây dựng đa thức nội suy Lagrange có nghĩa là xây dựng một
đường cong đại số bậc không quá n−1 đi qua tất cả các điểm (x i , yi ) với i = 1, 2, . . . , n.
Ví dụ, từ công thức của f0 (x), cho n = 2 ta được:
x − x2 x − x1
f0 (x) = c1 + c2 ,
x1 − x2 x2 − x1
với deg f0 (x) = 1, đây là phương trình của một đường thẳng đi qua hai điểm (x 1 , c1 ), (x 2 , c2 )
trong mặt phẳng tọa độ.
Cho n = 3, ta được
(x − x 2 )(x − x 3 ) (x − x 3 )(x − x 1 ) (x − x 1 )(x − x 2 )
f0 (x) = c1 + c2 + c3 ,
(x 1 − x 2 )(x 1 − x 3 ) (x 2 − x 3 )(x 2 − x 1 ) (x 3 − x 1 )(x 3 − x 2 )
với deg f0 (x) = 2, đây là phương trình của một parabol đi qua ba điểm (x 1 , c1 ), (x 2 , c2 ), (x 3 , c3 ).

2. Một số áp dụng của công thức nội suy Lagrange trong


giải toán

Ví dụ 1.1. Phân tích thành nhân tử

a2 (m − b)(m − c)(c − b) + b2 (m − c)(m − a)(a − c) + c 2 (m − a)(m − b)(b − a).

Lời giải. Xét đa thức f (x) = x 2 , áp dụng công thức nội suy Lagrange, ta có:
x −b x −c x −a x −c x −a x −b
f (x) = f (a) · + f (b) · + f (c) · .
a−b a−c b−a b−c c−a c−b
Suy ra
f (x)(a−b)(a−c)(b−c) = f (a)(x−b)(x−c)(b−c)+ f (b)(x−a)(x−c)(c−a)+ f (c)(x−a)(x−b)(a−b).
Cho x = m, ta được:
a2 (m−b)(m−c)(c−b)+b2 (m−c)(m−a)(a−c)+c 2 (m−a)(m−b)(b−a) = m2 (a−b)(a−c)(b−c).

Tập san Toán học STAR EDUCATION


5

Ví dụ 1.2. Cho đa thức f (x) = x 3 + ax 2 + bx + c trong đó a, b, c là các số thực. Chứng


3
minh rằng một trong các số sau | f (1)|, | f (2)|, | f (3)|, | f (4)| không nhỏ hơn . „
4

Lời giải. Áp dụng công thức nội suy Lagrange ta có:


(x − 2)(x − 3)(x − 4) (x − 1)(x − 3)(x − 4) (x − 1)(x − 2)(x − 3)
f (x) = f (1) + f (2) + · · · + f (4)
(1 − 2)(1 − 3)(1 − 4) (2 − 1)(2 − 3)(2 − 4) (4 − 1)(4 − 2)(4 − 3)
−1 1 1
= f (1)(x 3 − 9x 2 + 26x − 24) + f (2)(x 3 − 8x 2 + 19x − 12) − f (3)(x 3 − 7x 2 + 14x − 8)+
6 2 2
1
f (4)(x 3 − 6x 2 + 11x − 6)
6
f (1) f (2) f (3) f (4) 3
= (− + − + )x + · · ·
6 2 2 6
Đồng nhất hệ số của x 3 ta được
f (1) f (2) f (3) f (4)
1=− + − + .
6 2 2 6
Mặt khác, ta lại có:
f (1) f (2) f (3) f (4) 1 1 1 1
− + − + ≤ | f (1)| + | f (2)| + | f (3)| + | f (4)|
6 2 2 6 6 2 2 6
1 1 1 1
≤ ( + + + ) max{| f (1)|, | f (2)|, | f (3)|, | f (4)|}
6 2 2 6
4
≤ max{| f (1)|, | f (2)|, | f (3)|, | f (4)|}.
3
4
Do đó 1 ≤ max{| f (1)|, | f (2)|, | f (3)|, | f (4)|}.
3
4 3
Hay max{| f (1)|, | f (2)|, | f (3)|, | f (4)|} ≥ .
3 4

n
Y
Ví dụ 1.3. Cho x 0 , x 1 , . . . , x n là n + 1 số thực phân biệt. Đặt f (x) = (x − x i ). Khi
i=0
đó mỗi đa thức
P(x) = a0 x n + a1 x n−1 + · · · + an−1 x + an
n
X p(x i )
bậc n, ta có a0 = 0 (x )
. „

i=0
f i

n
Y x − xk 1 f (x)
Chứng minh. Vì = 0 nên ta suy ra
k6=i,k=0
xi − xk f (x) x − x i
n
X p(x i ) f (x)
p(x) = . .
i=0
f 0 (x i ) x − x i
n
X p(x i )
So sánh hệ số của x ta suy ra a0 =
n
.
i=0
f 0 (x i )

Tập san Toán học STAR EDUCATION


6

Ví dụ 1.4. Chứng minh rằng nếu x 1 , x 2 , . . . , x n là n nghiệm phân biệt của đa thức
n
X x is
f (x) thì 0 (x )
= 0 với 0 ≤ s ≤ n − 2. „

i=1
f i

Chứng minh. Do x 1 , x 2 , x 3 , . . . , x n là n nghiệm phân biệt của f (x) nên


n
X
f (x) = a (x − x i ), (a 6= 0).
i=1
n
X n
Y n
Y
Suy ra f (x) = a
0
(x − x j ) và f (x i ) = a
0
(x i − x j ) với 1 ≤ i ≤ n. Với
k=1 j=1, j6=k j=1, j6=i
0 ≤ s ≤ n − 2, xét đa thức g(x = x s ) và áp dụng khai triển Lagrange ta có
n n
X Y x − xi
g(x) = g(x i ) .
i=1 j=1, j6=i
x i − x j

Cân bằng hệ số của x n−1 ta được


n n n
X Y 1 X ax is
0= g(x i ) = .
i=1 j=1, j6=i
(x i − x j ) i=1
f 0 (x i )
n
X x is
Suy ra = 0.
i=1
f 0 (x i )

n  ‹
j n
X
Ví dụ 1.5. Chứng minh rằng (−1) j n = (−1)n n!.
j=0
j
„

Chứng minh. Xét đa thức f (x) = (x + x 1 )(x + x 2 ) . . . (x + x n ) với các x i tùy ý. Áp dụng
công thức nội suy Lagrange ta có
n n
X Y x −i
f (x) = f ( j) .
j=0 i=0,i6= j
j−i

Đồng nhất hệ số của x n ta được:


n n n
X Y 1 X 1
1= f ( j) = f ( j)
j=0 i=0,i6= j
j − i j=0
j( j − 1) · · · 1(−1)(−2) · · · ( j − n)
n n
1 1
 ‹
j n
X X
= f ( j) = (−1) f ( j).
j=0
(−1) n− j j!(n − j)! (−1) n n!
j=0
j
n  ‹
X n
Suy ra (−1) j f ( j) = (−1)n n!
j=0
j
Cho x 1 = x 2 = · · · = x n = 0 ta có f (x) = x n và do đó
n  ‹
j n
X
(−1) j n = (−1)n n!.
j=0
j

Tập san Toán học STAR EDUCATION


7

3. Bài tập luyện tập


Bài 1. Cho f (x) là đa thức bậc năm thỏa mãn f (1) = 1, f (2) = 1, f (3) = 2, f (5) =
5, f (6) = 8. Tính f (7).

Bài 2. Tính tổng

cos 1◦ cos 2◦ cos 3◦


S= + +
(cos 1◦ − cos 2◦ )(cos 1◦ − cos 3◦ ) (cos 2◦ − cos 1◦ )(cos 2◦ − cos 3◦ ) (cos 3◦ − cos 1◦ )(cos 3◦ − cos 1◦ )
n
n+ j
 ‹ ‹
j n
X
Bài 3. Chứng minh rằng (−1) = (−1)n .
j=0
j j

Bài 4. Cho x 0 , x 1 , . . . , x n là n + 1 số thực khác 0 và 1. Chứng minh rằng


n
X x in+1 f (x i−1 )
· = (−1)n+1 x 0 x 1 . . . x n ,
i=0
f 0 (x i ) 1 + xi

n
Y
trong đó f (x) = (x − x i ).
i=0

Bài 5. Cho f (x) là đa thức bậc ba với hệ số thực thỏa mãn

| f (1)| = | f (2)| = | f (3)| = | f (5)| = | f (6)| = | f (7)| = 12.

Tính | f (0)|.
i
Bài 6. Cho đa thức f (x) bậc n thỏa mãn f (i) = với i = 0, 1, 2, . . . , n. Tính
i+1
f (n + 1).

Bài 7. Cho f (x) là đa thức đơn khởi (hệ số cao nhất bằng 1) có 20 nghiệm phân biệt,
1
các nghiệm có dạng k với k = 0, 1, 2, . . . , 19. Tìm hệ số của x 18 trong khai triển của
3
P(x).

Tập san Toán học STAR EDUCATION


Tài liệu tham khảo

[1] Dương Quốc Việt (Chủ biên)-Đàm Văn Nhỉ, Cơ sở Lý thuyết số và đa thức, Nhà
xuất bản đại học sư phạm Hà Nội, 2012.

[2] Dương Quốc Việt (Chủ biên)-Đàm Văn Nhỉ, Giáo trình đại số sơ cấp, Nhà xuất bản
đại học sư phạm Hà Nội, 2016.

[3] Nguyễn Văn Mậu, Nội suy đa thức, định lý và áp dụng, Nhà xuất bản đại học Quốc
gia Hà Nội, 2016.

[4] Nguyễn Viết Đông-Trần Ngọc Hội Đại số đại cương, Nhà xuất bản đại học Quốc
gia TPHCM, 2011.

8
Đẳng giác - Liên hợp

Nguyễn Tăng Vũ

Bài viết này tôi đã viết cách đây 10 năm, lần này xin đăng lại để các em học sinh
có thể tham khảo thêm. Trong bài viết này ta tìm hiểu các khái niệm về đẳng giác:
đường đẳng giác, điểm đẳng giác và trường hợp đặc biệt là đường đối trung, điểm
humpty-dumpty. Đây cũng là một nội dung khá thú vị khi kiến thức đơn giản nhưng
cho ta nhiều mô hình, tính chất đẹp. Những bài toán liên quan đến đường đẳng giác,
điểm đẳng giác cũng hay xuất hiện trong các kì thi hình học trong và ngoài nước.

1. Đường đẳng giác - Điểm liên hợp

Định nghĩa 1. Cho xO y, hai đường thẳng d1 , d2 qua O đối xứng qua phân giác góc
xO y được gọi là hai đường đẳng giác ứng với xO y. Tức là (Ax; d1 ) = −(Ay; d2 )(
mod π).

Một số mô hình là hai đường đẳng giác thường gặp qua ví dụ sau, các bạn tự kiểm tra
lại nhé.
Tính chất 1. Một số đường đẳng giác đặc biệt
trong tam giác A
1. Đường phân giác thì đẳng giác với chính
nó.

2. Trong một tam giác vuông, đường cao O


và trung tuyến xuất phát từ đỉnh góc
vuông là đẳng giác.
B H C
3. Cho tam giác ABC nội tiếp đường tròn
tâm O. Khi đó đường AO và đường cao D
AH đẳng giác ứng với góc A.

9
10

Sau đây là một số tính chất quan trọng của hai đường đẳng giác.

Định lý 2. (Định lý Steiner) Cho tam giác ABC, các điểm D, E thuộc đường thẳng
BC. Khi đó AD, AE là hai đường đẳng giác đối với góc A khi và chỉ khi:

BD BE AB 2
. =
DC EC AC 2

Chứng minh. Điều kiện cần: Khi AD, AE là hai đường đẳng giác.

Ta có
BD SABD AD · AB · sin(AB; AD) AB sin(AB; AD)
= = = · (2.1)
DC SADC AD · AC · sin(AD; AC) AC sin(AD; AC)
Tương tự ta cũng có
BE AB sin(AB; AE)
= · (2.2)
EC AC sin(AE; AC)

∠(AB; AD) = ∠(AE; AC), ∠(AD; AC) = ∠(AB; AE) (2.3)
Từ (3.1), (3.2) và (3.3) ta có điều cần chứng minh.
Điều kiện đủ: Khi AD, AE thỏa (1), ta chứng minh AD, AE là hai đường đẳng giác.
Dựng AD0 đẳng giác với AE (D0 thuộc BC). Khi đó ta có hệ thức:
BD0 BE AB 2
. = .
D0 C EC AC 2
Kết hợp với (3.1) ta có
BD BD0
= ⇒ D ≡ D0 (2.4)
DC DC0

Vậy AD là đẳng giác với AE.

B D E C

Tính chất 2. Cho tam giác ABC, D, E thuộc cạnh BC và AD, AE đẳng giác đối với góc
A. Khi đó đường tròn ngoại tiếp các tam giác ADE và ABC tiếp xúc nhau.

Có một số cách để chứng minh đường hai đường thẳng đẳng, ngoài định nghĩa ta có
các tính chất sau.

Tập san Toán học STAR EDUCATION


11

Tính chất 3. Trong tam giác ABC, DE là đối song với BC (B, C, D, E đồng viên). Khi
đó đường trung tuyến, đường cao của xuất phát từ A của hai tam giác ADE và ABC là
các đường đẳng giác.

Tính chất 4. Cho góc xO y và đường thẳng d1 qua O. A là một điểm bất kì trên d1 , gọi
H, K lần lượt là hình chiếu của A trên Ox, O y. Khi đó đường thẳng d2 qua O và vuông
góc với H K khi và chỉ khi d2 là đường đẳng giác của d1 ứng với góc xO y.

Chứng minh. Ta chỉ xét hình như hình vẽ, các trường hợp khác chứng minh tương
tự.
Điều kiện cần: d2 là đường đẳng giác của d1 . Ta chứng minh d2 ⊥H K.
Ta có OHAK nội tiếp đường tròn đường kính OA, suy ra ∠AOH = ∠AK H.
Mà ∠KOB = ∠AOH, suy ra ∠KOB = ∠AK H.
Hơn nữa ∠AK H + ∠H KO = 90◦ , do đó ∠KOB + ∠H KO = 90◦ , suy ra OB⊥H K.
Điều kiện đủ: d2 qua O và vuông góc với K H.Ta chứng minh d2 là đường đẳng giác
của d1 .
Gọi đường thẳng d 0 là đường đẳng giác của d1 ứng với góc xO y, theo phần trên ta có
d 0 ⊥K H, suy ra d 0 trung d2 . Vậy d2 là đường đẳng giác của d1 .

A2

K
B
A1
H
y
x
A

Tập san Toán học STAR EDUCATION


12

Tính chất 5. Gọi A1 , A2 là điểm đối xứng của A qua Ox và O y. Khi đó đường trung
trực của A1 A2 là đường đẳng giác của OA.

Sau đây là một định lý quan trọng và có nhiều áp dụng.

Tính chất 6. Cho góc xO y, A và B là hai điểm sao cho OA, OB là hai đường đẳng giác
ứng với xO y. Gọi A1 , A2 là hình chiếu của A trên Ox, O y; B1 , B2 là hình chiếu của B
trên O x, O y. Khi đó bốn điểm A1 , A2 , B1 , B2 cùng thuộc một đường tròn và tâm đường
tròn là trung điểm của đoạn AB.

B1 B2
A2
B

A1 I
x y
A

Chứng minh. Ta có OA1 = OA · cos AOA1 và OB1 = OB · cos BOB1 .


Và OA2 = OA · cos AOA2 và OB2 = OB · cos BOB2 .
Với tính chất đẳng giác, ta có OA1 · OB1 = OA2 · OB2 . Do đó 4 điểm A1 , A2 , B1 và B2
cùng thuộc một đường tròn. Hơn nữa tâm chính là trung điểm của AB.

Tính chất 7. Cho góc Ox y, A, B thỏa OA, OB là đẳng giác với Ox y. Gọi H, K là hình
chiếu của A trên Ox và của B trên O y, M là giao điểm của BH, AK, khi đó OM vuông
góc với AB.

Tính chất 8. Cho góc Ox y, OA, OB là đẳng giác với Ox y. Gọi A1 , A2 là hình chiếu
của A trên Ox, O y; B1 , B2 là hình chiếu của B trên Ox, O y. Gọi K, L là giao điểm của
OB, AB với A1 A2 . Khi đó 4 điểm B1 , B2 , K, L đồng viên.

Tập san Toán học STAR EDUCATION


13

A2
D L
A1 K

B1 A B2
E
P I
x y
B

Lời giải. Gọi D là giao điểm của OA và A1 A2 , E là giao điểm của OB và B1 B2 , P là giao
điểm của A1 A2 và B1 B2 .Dễ thấy các cặp tam giác ODA1 , OEB2 và OAA1 , OBB2 đồng
OD OA1 OA
dạng, suy ra = = , suy ra E D // AB.
OE OB2 OB
Tam giác OP E có OA⊥P E, P L⊥OE nên D là trực tâm, suy ra DE⊥OP, do đó AB⊥OP
tại T .Suy ra P T ⊥PO = P K · P L.
Hơn nữa OTA1 A2 và A1 A2 B2 B1 nội tiếp nên P T · PO = PA1 · PA2 = P B1 · P B2 .
Khi đó P K · P L = P B1 · P B2 , suy ra B1 , K, L, B2 đồng viên.

Tính chất 9. Cho tam giác ABC. Các cặp đường thẳng da , da0 là đường đẳng giác ứng
với góc A; định nghĩa tương tự với các cặp d b , d b0 và dc , dc0 . Khi đó da , d b , dc đồng quy
(tại P) khi và chỉ khi các đường thẳng da0 , d b0 , dc0 đồng quy tại Q.

Chứng minh. Giả sử các đường thẳng da , d b , dc đồng quy. Theo định lý Ceva dạng sin
ta có:
sin (da , c) sin (d b , a) sin (dc , b)
. . = −1
sin (da , b) sin (d b , c) sin (dc , a)
sin (da , c) sin (d 0 a , b)
Mà (da , c) = (−da0 , b), (da , b) = −(da0 , c) suy ra = .
sin (da , b) sin (d 0 a , c)
sin (d b , a) sin (d 0 b , c) sin (dc , b) sin (d 0 c , a)
Tương tự thì: = ; =
sin (d b , c) sin (d 0 b , a) sin (dc , a) sin (d 0 c , b)
Từ đó:
sin (d 0 a , b) sin (d 0 b , c) sin (d 0 c , a)
. . = −1
sin (d 0 a , c) sin (d 0 b , a) sin (d 0 c , b)
Vậy da0 , d b0 , dc0 đồng quy.

Tập san Toán học STAR EDUCATION


14

Tính chất 10. Cho tam giác ABC, nếu AP, AQ đẳng giác và BP, BQ đẳng giác thì C P, CQ
cũng đẳng giác.

Ta chứng minh tính chất này cũng dựa vào định lý trên.

Định nghĩa 2. Trong một tam giác hai điểm được gọi là hai điểm liên hợp đẳng giác
hay còn gọi là liên hợp đối với một tam giác nếu các cặp đường thẳng qua mỗi đỉnh
và hai điểm đó là các cặp đường thẳng đẳng giác.

Việc phát hiện được hai điểm liên hợp trong một tam giác khá quan trọng trong việc
giải quyết các bài toán, ta có thể xem một số trường hợp sau.

Nhận xét. Trong một tam giác tâm đường tròn nội tiếp và tâm các đường tròn bàng tiếp
là liên hợp với chính nó.

Nhận xét. Ta thấy trong một tam giác trực tâm và tâm đường tròn ngoại tiếp là hai
điểm liên hợp. Nên hình chiếu của trực tâm trên các cạnh và hình chiếu của tâm ngoại
tiếp trên các cạnh, do đó các hình chiếu này đồng quy. Đây chính là định lý về đường
tròn Euler, tâm đường tròn Euler theo định lý trên là trung điểm của đoạn thẳng nối trực
tâm và tâm ngoại tiếp.

Định lý 3. Trong một tam giác, hình chiếu của hai điểm liên hợp trên các đường
thẳng chứa cạnh của tam giác cùng thuộc một đường tròn. Đường tròn này tâm là
trung điểm của đoạn thẳng nối hai điểm liên hợp.

Định nghĩa 3. Cho tam giác và một điểm bất kì, khi đó các điểm đối xứng của điểm
này qua các đường thẳng chứa cạnh tam giác tạo thành một tam giác, được gọi là
tam giác đối xứng.

Tính chất 11. Cho tam giác ABC, P là điểm nằm trong tam giác. Gọi D, E, F lần lượt
là điểm đối xứng của P qua BC, AC, AB. Khi đó đường thẳng qua vuông góc E F , đường

Tập san Toán học STAR EDUCATION


15

thẳng qua B vuông góc DF và đường thẳng qua C vuông góc DE đồng quy tại một
điểm. Điểm này chính là điểm liên hợp của P.

Định lý này có thể chứng minh dễ dàng dựa vào tính chất 4.3.

Trên đây là các tính chất quan trọng của hai đường đẳng giác và hai điểm liên hợp. Có
nhiều bài toán việc xác định các đường đẳng giác hay các điểm liên hợp là rất quan
trọng, là chìa khóa để ta giải được bài toán. Ta bắt đầu với các ví dụ đơn giản nhất.

Ví dụ 2.1. Cho tam giác ABC, một đường tròn thay đổi luôn đi qua B, C cắt AB, AC
tại D, E. Chứng minh rằng tâm đường tròn ngoại tiếp tam giác ADE luôn thuộc một
đường cố định. „

Lời giải. Gọi O là tâm đường tròn ngoại tiếp tam giác ABC và I là tâm của đường tròn
ngoại tiếp tam giác ADE.
Ta có ADE và AC B đồng dạng, nên AI D và AOC cũng đồng dạng, suy ra AI và AO là
hai đường đẳng giác đối với góc ∠BAC.
Mặt khác đường cao AH và AO cũng là hai đường đẳng giác, suy ra AH trùng AI hay
I thuộc AH cố định.

Ví dụ 2.2. Cho tam giác ABC gọi I a là tâm đường tròn bàng tiếp ứng với A, da là
đường thẳng qua I a vuông góc với BC; các đường thẳng d b , dc được xác định tương
tự. Chứng minh rằng da , d b , dc đồng quy. „

Lời giải. Ta có I a A, I b B, I c C là các đường cao của tam giác I a I b I c .


Ta có BC và I b I c là đối song nên da và I a A là đẳng giác. Do đó da , d b , dc đồng quy.

Ví dụ 2.3. Cho tam giác ABC, D khác phía C đối với AB và E khác phía B đối với AC
sao cho ∠ABD = ∠AC E = 90◦ , ∠DAB = ∠CAE. Gọi M là trung điểm DE.

1. Chứng minh M BC cân.

2. Gọi H là hình chiếu của A trên DE. Chứng minh B, C, H, M cùng thuộc một
đường tròn.

Lời giải.

1. Ta thấy AB, AC đẳng giác ứng với góc ∠DAE, do đó trung điểm M của DE cách
đều B và C hay tam giác M BC cân tại M .

2. Các tứ giác ADBH, AEC H nội tiếp nên ∠BH D = ∠DAB = ∠EAC = ∠C H E, suy
ra H E là phân giác ngoài của ∠BH C.
Ta có H M là phân giác ngoài của ∠BH C và M B = M C nên B, C, M , H đồng
viên.

Tập san Toán học STAR EDUCATION


16

Ví dụ 2.4. Cho tam giác ABC, gọi D, E là hình chiếu của B, C trên phân giác trong
góc A. Đường tròn đường kính DE cắt BC tại M , N . Chứng minh rằng đường tròn
ngoại tiếp của tam giác AM N và ABC tiếp xúc nhau. „

Lời giải. Gọi P, Q là giao điểm của (AM N ) với AB, AC.
Ta có BD2 = BM · BN = BP · BA, C E 2 = C M · C N = CQ · CA
BP · BA BD2
Suy ra = .
CQ · CA C E 2
BD AB BP AB
Mà = nên = ⇒ PQ // BC.
CE AC CQ AC
Khi đó ∠BAM = ∠PQM = ∠QP N = ∠CAN , từ đó có AM , AN đẳng giác với ∠BAC,
suy ra (AM N ), (ABC) tiếp xúc nhau.

Ví dụ 2.5. Cho tam giác ABC nhọn có AB > BC, AC > B. Trên cạnh AB, AC lấy M , N
sao cho C M = BC = BN . Gọi H là trực tâm tam giác ABC, chứng minh tâm đường
tròn ngoại tiếp tam giác H M N thuộc đường thẳng Euler của tam giác ABC. „

A ∠OC N . Tam giác H C N , OBC cân và


∠OC B = ∠H C N , suy ra 4OC B ö
CN CH CH
4N C H.Suy ra = = .
CO CB CM
Do đó 4COH ö 4C M N , suy ra ∠C HO =
O N ∠C N M
P
Và ∠OHQ = ∠OH C − ∠C HQ = ∠C N M −
I
M Q ∠P H N = ∠C N M − ∠N M A = ∠BAC =
∠N H E.
H Mặt khác I là tâm đường tròn ngoại tiếp
B C tam giác H NQ và H E⊥EQ.
Suy ra ∠I HQ = ∠N H E = ∠OHQ. Vậy
Lời giải. Ta có ∠M C H = ∠BC H = H, I, O thẳng hàng.

Ví dụ 2.6. Cho tam giác ABC có tâm ngoại tiếp là O, gọi A1 , B1 , C1 là trung điểm của
BC, AC, AB. Các đường cao AD, BE, C F , gọi A0 là giao điểm của OA và E F , các điểm
B 0 , C 0 được xác định tương tự. Chứng minh rằng (A1 B1 C 0 ), (B1 C1 A0 , (A1 C1 B 0 ) cùng đi
qua một điểm thuộc đường thẳng Euler của tam giác ABC. „

Tập san Toán học STAR EDUCATION


17

A0 E
T
F
H B1
C1
K
O
C0

B D A1 C
S

Lời giải. Bước 1: Gọi T, K là giao điểm của OH với E F và DE.


Ta có AH, AO đẳng giác với ∠BAC, E, F là hình chiếu của H trên AC, AB và B1 , C1 là
hình chiếu của O trên AC, AB, suy ra T thuộc (B1 C1 A0 ).
Tương tự K thuộc A1 B1 C 0 ).
Gọi S là giao điểm của (B1 C1 A0 ) với OH khác T . Ta chứng minh S thuộc (A1 B1 C 0 ).
Thật vậy ta có ∠B1 SK = ∠B1 A0 E = ∠B1 C 0 E = ∠B1 A1 K, suy ra S thuộc (A1 B1 K) hay S
thuộc (A1 B1 C 0 ).
Tương tự cũng chứng minh được S thuộc (A1 C1 B 0 ).
Do đó (A1 B1 C 0 ), (B1 C1 A0 , (A1 C1 B 0 ) cùng đi qua S thuộc đường thẳng Euler của tam
giác ABC

Tập san Toán học STAR EDUCATION


18

Trên đây là một ví dụ áp dụng tính chất, trực tâm và tâm ngoại tiếp là hai điểm liên
hợp. Đây là hai điểm liên hợp quan trọng và quen thuộc trong tam giác. Sau đây ta
xét một vài cặp điểm liên hợp khác qua các ví dụ sau.

Ví dụ 2.7. Cho tam giác ABC. Gọi O là tâm đường tròn ngoại tiếp tam giác ABC;
O1 , O2 , O3 lần lượt là tâm ngoại tiếp các tam giác BCO, ACO và ABO. Chứng minh
rằng AO1 , BO2 , CO3 đồng quy tại một điểm, điểm này là điểm liên hợp với tâm đường
tròn Euler của tam giác ABC. (Điểm Kosnita) „

Lời giải. Gọi M là trung điểm BC và P là A


giao điểm hai tiếp tuyến tại B, C. Khi đó
Oa là trung điểm của OP. Gọi K là điểm
đối xứng của O qua BC, thì AK đi qua tâm
đường tròn Euler. Ta chứng minh AOa và H O
AK đẳng giác với ∠BAC.
Dễ thấy OM · OP = OB 2 = OA2
Mà OK = 2OM , OP = 2OOa , suy ra OM · B M C
OP = OK · OOa
Khi đó OA2 = OK · OOa , khi đó tam giác K
OAK và OOa A đồng dạng, suy ra ∠OAK =
∠OAOa .
Oa
Hơn nữa ∠OAOa = ∠HAOa nên ∠OAK =
∠HAOa
Khi đó ta có AK, AOa đẳng giác với ∠HAO
nên cũng đẳng giác với ∠BAC.

Ví dụ 2.8. Cho tam giác ABC nội tiếp đường tròn (O), đường cao AD. Tiếp tuyến tại
B, C cắt nhau tại P, gọi H, K là hình chiếu của P trên các đường thẳng AB và AC. Gọi
M là trung điểm BC, đường tròn (M H K) cắt BC tại điểm thứ hai là N . Chứng minh
C N = BD. „

Lời giải. Gọi Q là điểm đối xứng của A qua M , khi đó ABQC là hình bình hành. Suy
ra ∠QBC = ∠BAC = ∠P BH, ∠QC B = ∠ABC = ∠QC K. Từ đó P, Q là liên hợp đối với
tam giác ABC.
Khi đó áp dụng định lý 3.2 thì hình chiếu của P, Q trên các đường thẳng AB, AC, BC
cùng thuộc một đường tròn, do đó QN ⊥BC. Do tính chất đối xứng, suy ra C N = BD.

Tập san Toán học STAR EDUCATION


19

M N C
B D K

H
P
Q

Ví dụ 2.9. (Sharygin 2010) Đường tròn nội tiếp tam giác nhọn ABC tiếp xúc với các
cạnh AB, BC, AC lần lượt là C1 , A1 , B1 . Các điểm A2 , B2 lần lượt là trung điểm các đoạn
B1 C1 , A1 C1 . Gọi P là giao điểm của đường tròn nội tiếp và CO, với O là tâm đường
tròn ngoại tiếp tam giác ABC. Gọi N , M là giao điểm thứ hai của PA2 , P B2 với đường
tròn nội tiếp. Chứng minh rằng giao điểm của AN và BM thuộc đường cao hạ từ C
của tam giác ABC. „

Lời giải. Gọi I là tâm đường tròn nội tiếp tam giác ABC và K là giao điểm của AN và
BM . Ta có A2 I ·A2 A = A2 C1 ·A2 B1 và A2 C1 ·A2 B1 = A2 N ·A2 P suy ra A2 N ·A2 P = A2 I ·A2 A,
do đó AN I P nội tiếp. Mà I N = I P nên AI là phân giác ∠N AP. Từ đó AN , AP là đẳng
giác đối với góc A.
Chứng minh tương tự ta có BM , BP đẳng giác với góc B.
Mà AP, BP, CO đông quy tại P; AN , BM cắt nhau tại K, khi đó C K đẳng giác của CO,
suy ra K thuộc đường cao hạ từ C của tam giác ABC.

Ví dụ 2.10. (VN TST 2015) Cho tam giác ABC nhọn và có một điểm P nằm trong
tam giác sao cho ∠AP B = ∠AP C = α và ∠α > 180o − ∠BAC. Đường tròn ngoại tiếp
tam giác AP B cắt AC tại E; đường tròn ngoại tiếp tam giác AP C cắt AB tại F . Gọi Q là
điểm nằm trong tam giác AE F sao cho ∠AQE = ∠AQF = α. Gọi D là điểm đối xứng
của Q qua đường thẳng E F . Phân giác góc ∠E DF cắt AP tại điểm T .

1. Chứng minh rằng ∠DE T = ∠ABC và ∠AF T = ∠AC B.

2. Đường thẳng AP cắt các đường thẳng DE và DF tại M , N . Gọi I là tâm đường
tròn nội tiếp tam giác E P M và J là tâm đường tròn nội tiếp tam giác F P N . Gọi
K là tâm đường tròn ngoại tiếp tam giác DI J. DT cắt (K) tại H. Chứng minh
đường thẳng K H đi qua tâm đường tròn nội tiếp tam giác DM N .

Lời giải.
1. Do ∠AP B > ∠180o − ∠BAC nên E thuộc tia đối của tia AC, tương tự F thuộc tia
đối của tia AB.

Tập san Toán học STAR EDUCATION


20

Ta có ∠E P B = ∠EAB = ∠FAC = ∠F P C. Suy ra ∠AP E = ∠AF E. Suy ra ∠ABE =


∠AC F . Do đó tứ giác BE F C nội tiếp.
Suy ra ∠ABC = ∠AE F, ∠AC B = ∠AF E. Tam giác ABC và AE F đồng dạng nghịch
hướng.
Từ đó ta có AQE và AP B đồng dạng, AQC và AP C đồng dạng.
AT, AQ đẳng giác đối với góc EAF .
Gọi T 0 là điểm đẳng giác của Q đối với tam giác AE F , ta có T 0 ∈ AP.
Khi đó ∠T 0 E F = ∠QEA = ∠AE P, suy ra ∠T 0 E D = ∠T 0 E P, tương tự thì ∠T 0 F P =
∠T 0 F P.
Áp dụng định lý Ceva sin cho tam giác DE F ta có:

sin ∠T 0 DE sin ∠T 0 F D sin ∠T 0 E F sin ∠T 0 DF sin ∠T 0 F D sin ∠T 0 E F


. . = 1 ⇒ = .
sin ∠T 0 DF sin ∠T 0 F E sin ∠T 0 E D sin ∠T 0 DE sin ∠T 0 F E sin ∠T 0 E D

Áp dụng định lý Ceva sin cho tam giác P E F ta có:

sin ∠T 0 P E sin ∠T 0 F P sin ∠T 0 E F sin ∠T 0 F P sin ∠T 0 E F


. . = 1 ⇒ . . =1
sin ∠T 0 P F sin ∠T 0 F E sin ∠T 0 E P sin ∠T 0 F E sin ∠T 0 E P

Từ đó ta có ∠T 0 DE = ∠T 0 DF . Suy ra DT 0 là phân giác góc ∠DE F , vậy T 0 ≡ T .


Do đó ∠DE T = ∠T E P = ∠AE F = ∠ABC và ∠E F T = ∠AC B.

2. Áp dụng Ceva sin cho tam giác DE F với điểm T và tam giác T E F với điểm P ta
chứng minh được T N và T D là hai đường đẳng giác đối với ∠E T F .
Ta có I ∈ E T, J ∈ F T .
Gọi X là tâm đường tròn nội tiếp tam giác DM N , U là giao điểm của M I và DT .
UI TU
Khi đó ∆T U I ∼ T N J, suy ra =
NJ TN
TU DU
Mà = (do NU là phân giác ∠DN T ).
TN DN
UI DU
Suy ra = Mặt khác ta có ∠DU I = ∠DN J, suy ra ∆DU I ∼ ∆DN J, suy
NJ DN
ra ∠DI U = ∠DJ N , suy ra DX J I nội tiếp. Mà ∠X DH = 90o nên X H là đường
kính, hay X H qua K.
Vậy K H qua tâm đường tròn nội tiếp tam giác DMN.

Tập san Toán học STAR EDUCATION


21

2. Đường đối trung - Điểm Lemoine

Định nghĩa 4. Trong một tam giác đường đẳng giác với trung tuyến xuất phát từ một
đỉnh được gọi là đường đối trung của tam giác.

Tính chất 12. Cho tam giác ABC, D là điểm thuộc cạnh BC, H, K là hình chiếu của
D trên các đường thẳng AB, AC. Khi đó các mệnh đề sau tương đương.

1. AD là đường đối trung.

DB AB 2
2. = .
DC AC 2
sin DAB AB
3. = .
sin DAC AC
DH AB
4. = (H, K lần lượt là hình chiếu của D trên AB, AC).
DK AC

B D M C

Sau đây là một số tính chất để nhận biết được đường đối trung. Ta có thể chứng minh
dễ dàng dựa và tính chất 4.20.

Tính chất 13. Cho tam giác ABC, tiếp tuyến tại B và C của đường tròn ngoại tiếp tam
giác ABC cắt nhau tại P. Khi đó AP là đường đối trung của tam giác ABC.

Tính chất 14. Cho tam giác ABC, đường thẳng qua B vuông góc với AC cắt AB tại H;
đường thẳng vuông góc với AB tại B cắt AC tại K. Khi đó trung điểm H K thuộc đường
đối trung của tam giác ABC.

Tập san Toán học STAR EDUCATION


22

B C
D M

P
H

Tính chất 15. Các đường đối trung giao nhau tại một điểm, được gọi điểm Lemoine.
Điểm Lemoine và trọng tâm là hai điểm đẳng giác.

Tính chất 16. Trong một tam giác, điểm đẳng giác của trọng tâm được gọi là điểm
Lemoine. Chứng minh rằng điểm Lemoine là trọng tâm của tam giác Pedal tương ứng.

Chứng minh. Gọi P là giao tiếp tuyến của (O) tại B, C, thì AP là đường đối trung.
Gọi H 0 , L 0 là hình chiếu của P trên AB, AC. Ta chỉ cần chứng minh P M qua trung điểm
của H 0 L 0 , ta chứng minh P H 0 M L 0 là hình bình hành.
Thực vậy, trước hết ∠P H 0 M = ∠P BM = ∠P C M = ∠P L 0 M
Và ∠H 0 M L 0 = ∠H 0 M P +∠L 0 M P = ∠P BH 0 +∠P C L 0 = ∠AC B+∠ABC = 180◦ −∠BAC =
∠H 0 P L 0 .
Vậy H 0 M L 0 P là hình bình hành, suy ra điều cần chứng minh.

Sau đây ta xét một vài ví dụ liên quan đến đường đối trung.

Ví dụ 2.11. Cho tam giác ABC, một đường tròn thay đổi qua BC cắt các cạnh AB, AC
lần lượt tại D và E. Tiếp tuyến tại D và E của đường tròn ngoại tiếp tam giác ADE
cắt nhau tại P. Chứng minh rằng P luôn thuộc một đường thẳng cố định. „

Lời giải. Nhận xét P thuộc đường đối trung của tam giác ADE. Mà BC là đường đối
song của DE nên trung tuyến AM của tam giác ABC là đường đối trung của tam giác
ADE. Do đó P thuộc AM cố định.

Tập san Toán học STAR EDUCATION


23

Ví dụ 2.12. Cho tam giác ABC, đường thẳng song song với BC cắt AB, AC tại M , N .
Gọi P là giao điểm của BN và C M ; đường tròn ngoại tiếp các tam giác BM P và C N P
cắt nhau tại Q khác P. Chứng minh rằng ∠BAP = ∠CAQ. „

A Lời giải. Dễ thấy AP là trung tuyến của


tam giác ABC(Bổ đề hình thang). Ta
chứng minh AQ là đối trung của tam giác
ABC.
Ta có ∠MQC = ∠MQP+∠PQC = ∠ABP+
∠AN P = 180◦ −∠A, suy ra AMQC nội tiếp,
mặt khác ta có tam giác QM B, QN C đồng
M N
dạng. Từ đó
P
sin QAM QM BM AB sin PAB
= = = =
B sin QAC QC CN AC sin PAC
C
Q Do đó ∠QAB = ∠CAP.

Ví dụ 2.13. Cho tam giác ABC nhọn khác tam giác cân. M là trung điểm cạnh BC.
Gọi D, E là các điểm thuộc AM sao cho AD = BD, AE = EC. BD cắt C E tại F . Một
đường tròn qua B, C cắt các cạnh AB, AC tại H, K. Chứng minh rằng AF đi qua trung
điểm của H K. „

Lời giải. Ta thấy rằng H K là đường đối song của BC nên để chứng minh AF qua trung
điểm của H K thì ta chỉ cần chứng minh AF là đường đối trung của tam giác ABC. Áp
dụng định lý sine cho tam giác ABF và tam giác AC F , ta có:
AB sin AF B sin AF B AC sin AF C sin AF C
= = (1) và = = . (2)
AF sin ABF sin BAD AF sin AC F sin EAC
sin DAB AC
Mà D, E thuộc trung tuyến AM nên ta có: = . (3)
sin EAC AB
Từ (1), (2) và (3), ta suy ra sin AF B = sin AF C, tức ∠AF B = ∠AF C. (4)
Mặt khác ta lại có:

∠BF C = ∠F DE + ∠F E D = 2∠BAD + 2∠EAC = 2∠BAC = ∠BOC

Kết hợp với trên, ta được: ∠AF B = ∠AF C = 180◦ − ∠BAC.


Như vậy, ta có
∠FAC + ∠F CA = ∠BAC = ∠BAD + ∠CAD
, mà ∠F CA = ∠CAD nên ∠FAC = ∠BAD.
Vậy AF là đường đối trung của tam giác ABC, ta có điều cần chứng minh.

Tập san Toán học STAR EDUCATION


24

Ví dụ 2.14. Cho đường tròn (O) có dây BC cố định khác đường kính và điểm A di
động trên (O) sao cho tam giác ABC nhọn, không cân. Gọi I là trung điểm của BC và
D là trung điểm cung BC không chứa A của Đường thẳng qua K vuông góc với AO cắt
AB, AC lần lượt ở E, F . Đường thẳng qua A vuông góc với BC cắt K D tại G. Chứng
minh rằng đường trung tuyến đỉnh A của tam giác AE F chia đôi đoạn thẳng DG. „

T
A

O
F
B C

K
P
D

Q
E

Lời giải. Gọi Q là giao điểm của AP và DG, ta chứng minh QD = QG.
Ta có E F là đối song của BC nên trung tuyến của AP của tam giác AE F là đẳng giác
của trung tuyến AI của tam giác ABC, từ đó ta có AP, AK đẳng giác. Mặt khác có AG
và AO đẳng giác, từ đó suy ra AD là phân giác của ∠KAP và ∠OAG.
Vẽ đường kính DT của (ABC), khi đó AT ⊥AD nên OK⊥AT , suy ra OK là trung trực
của AT , do đó ∠K T D = ∠KAO = ∠QAG.
Từ đó suy ra 4K T D ö 4QAG, suy ra

KD KT AK DK
= = =
QG AQ AQ DQ

do đó QG = QD, ta có điều cần chứng minh.

Tập san Toán học STAR EDUCATION


25

3. Điểm Humty - Điểm Dumty

Định nghĩa 5. Trong tam giác ABC, hình chiếu của trực tâm tam giác trên trung tuyến
AM được gọi là điểm A − humpt y; tương tự ta cũng có các điểm B − humpt y, C −
humpt y.

Một số tính chất có thể chứng minh dễ dàng từ định nghĩa trên của các điểm humpty.

Tính chất 17. Gọi Pa là điểm A − humpt y của tam giác ABC. Khi đó ta có các tính
chất sau:

1. Pa thuộc đường tròn ngoại tiếp tam giác BH C.

2. ∠PAB = ∠P BC, ∠P BC = ∠P CA( mod π).

3. BC là tiếp tuyến chung của đường tròn ngoại tiếp các tam giác PAB, P BC.
1 2
4. A1 là trung điểm BC thì A1 A · A1 Pa = BC .
4

K1 B0

H Pa
C0

S B A0 A1 C

Chứng minh. Tam giác ABC có các đường cao AA0 , BB 0 , C C 0 cắt nhau tại H. Ta xét
tam giác ABC nhọn và Pa nằm trong tam giác.

1. Ta có các tứ giác BA0 H C 0 , A0 H Pa A1 nội tiếp, suy ra A1 BC 0 Pa nội tiếp, từ đó


∠A1 Pa B = ∠A1 C B = ∠A1 BC. Tương tự thì ∠A1 PaC = ∠A1 C B 0 .
Khi đó ∠BPa C == ∠A1 Pa B + A1 Pa C = ∠A1 BC 0 + ∠A1 C B 0 = 180◦ − ∠A = ∠BH C.
Do đó BH Pa C nội tiếp.

2. Ta có ∠Aa Pa B = ∠A1 BA, suy ra 4A1 Pa B ö 4A1 BA, suy ra ∠A1 AB = ∠A1 BPa và
1
Aa A · A1 Pa = A1 B 2 = BC 2 .
4
Chứng minh tương tự ta cũng có ∠A1 C Pa = ∠A1 AC.

3. Từ ý b ta có điều cần chứng minh.

Tập san Toán học STAR EDUCATION


26

4. Suy ra từ ý b.

Nhận xét. Nếu A là góc tù, thì ta đổi vị trí của A và H trong mô hình trực tâm, khi đó
Pa là hình chiếu của A trên đường thẳng HA1 , gọi là điểm K1 .

Định nghĩa 6. Điểm đẳng giác của điểm A− humpt y được gọi là điểm A− dumpt y),
đặt là Q a .

Ta có một số tính chất sau.

Tính chất 18. Cho tam giác ABC nội tiếp đường tròn w, với điểm A − humpt y và
A − dumpt y là Pa , Q a .

1. ∠Q a AC = ∠Q a BA, ∠Q a AB = ∠Q a CA.

2. Đường tròn ngoại tiếp tam giác Q a AB tiếp xúc với AC và đường tròn ngoại tiếp
tam giác Q a AC tiếp xúc với AB.

3. Điểm đối xứng của Pa qua BC thuộc w.

4. Đối xứng của A qua Q a thuộc w.

A 2. Suy ra từ a.

3. Đường đối trung đỉnh A cắt w tại D,


ta chứng minh BC là trung trực của
Pa D.
Pa Rõ ràng ∠Pa BC = ∠Pa AB =
∠DAC = ∠C BD và ∠Pa A1 B =
∠DA1 B. Từ đó 4Pa BA1 = 4DBA1 ,
Qa suy ra BPa = BD và A1 Pa = A1 D, do
C đó BC là trung trực của Pa D.
B A1
4. Rõ ràng Q a thuộc AD, ta chứng
minh Q a là trung điểm của AD.
D Từ các cặp tam giác ABQ a , CAQ a
đồng dạng và DBQ a , C DQ a đồng
dạng ta có AQ2a = Q a B·Q a C = Q a D2 ,
1. Suy ra từ tính chất 4.26 b và Q a liên suy ra AQ a = DQ a , hay Q a là trung
hợp với Q a và Pa . điểm AD.

Tập san Toán học STAR EDUCATION


27

Ta xét một bài toán trong các đề thi học sinh giỏi.

Ví dụ 2.15. Cho tam giác ABC, đường tròn w qua B tiếp xúc với AC tại A; đường tròn
w0 qua C tiếp xúc với AB tại A; gọi D là giao điểm thứ hai của w và w0 , AD cắt (ABC)
tại E. Chứng minh D là trung điểm của AE. „

Lời giải. D chính là điểm A − dumt y, theo tính chất 4.29 ta có thể chứng minh dễ
dàng.

Ví dụ 2.16. Cho tam giác ABC có tâm ngoại tiếp là O và trực tâm H. Đường tròn
đường kính AO cắt (BOC) tại M , AM cắt (BOC) tại X . Đường tròn đường kính AH
cắt (BH C) tại N , AN cắt (BH C) tại Y . Chứng minh M N song song với X Y . „

A Lời giải. Ta có N là giao điểm của đường


tròn đường kính AH và (BH C) nên N
là điểm A − humpt y, tương tự thì M là
A − dumpt y.
O Ta có ∠ABN = ∠M BC = ∠AX C và
M
∠N AB = ∠X AC nên 4ABN ö 4CAX , do
N đó AN · AX = AB · AC. Chứng minh tương
H
tự ta có AM · AY = AB · AC.
B C AM AN
Từ đó AM · AY = AN · AX ⇒ = , ta
AX AY
có M N // X Y .

Tập san Toán học STAR EDUCATION


28

Ví dụ 2.17. Cho lục giác AP BCQ nội tiếp đường tròn w. Một điểm M nằm trong tam
giác ABC thỏa

∠M AB = ∠M CA, ∠BAC = ∠M BA, ∠P M B = ∠QM C = 90◦

Chứng minh rằng BP, AM và CQ đồng quy. „

Q
A
P

O
M
B C

Lời giải. Gọi E là giao điểm của C P và AM .


Dễ thấy M là A − dumpt y của tam giác ABC, do đó OM ⊥AM và M thuộc đường tròn
ngoại tiếp tam giác BOC. Khi đó

1
∠BM E = 90◦ + ∠OBC = 180◦ − ∠BOC = 180◦ − ∠BP E
2

từ đó ta có BP E M nội tiếp.
Gọi Q0 là giao điểm của BE và (w), khi đó ∠C M E = ∠E M B = 180◦ − ∠C P B =
180◦ − ∠CQ0 E, suy ra M EQ0 C nội tiếp, từ đó ∠Q0 M C = ∠Q0 EC = 90◦ , do đó Q0 ≡ Q.
Khi đó xét 3 đường tròn (BP E M ), (C M EQ), w trục đẳng phương đồng quy, ta có điều
cần chứng minh.

Ví dụ 2.18. (Iran TST 2018): Cho tam giác ABC nhọn (AB < AC) với BE, C F là hai
đường cao. Gọi M , N lần lượt là giao điểm của đường phân giác trong của góc A với
E F, BC; P là điểm thỏa mãn P M ⊥ E F, P N ⊥ BC. Chứng minh rằng: AP đi qua trung
điểm của BC. „

Tập san Toán học STAR EDUCATION


29

E
M
E Q
H P

X
S B N C

Lời giải. Gọi S là giao điểm của E F và BC, Q là điểm A− humpt y. Ta có S, H, Q thẳng
hàng và Q thuộc (AE F ).Do AQ là đối trung của tam giác AE F nên QM đi qua điểm
chính giữa cung E F của (AE F ), khi đó

1 1 1
∠HQM = 90◦ − (∠AE F −∠AF E) = 90◦ − (∠ABC +∠AC B) = ∠AC B + ∠A = ∠QN S
2 2 2

Do đó tứ giác SMQN nội tiếp, mặt khác SP là đường kính của (SM N ) nên SQ⊥QP,
suy ra A, Q, P thẳng hàng. Từ đó AP qua trung điểm của BC.

Ví dụ 2.19. Cho tam giác nhọn ABC nội tiếp (O). Lấy P và Q thuộc cạnh BC sao cho
PAB = BCA; CAQ = ABC. Lấy M thuộc AP, N thuộc AQ sao cho P là trung điểm của
AM và Q là trung điểm của AN. Chứng minh rằng: BM và CN cắt nhau tại một điểm
nằm trên (O). „

Lời giải. Gọi X là giao điểm của (ABP) và (ACQ), ta thấy X là điểm A − dumpt y. Xét
phép vị tự tâm A tỉ số 2 thì P 7→ M , Q 7→ N , X 7→ Y , trong đó Y thuộc đường tròn
ngoại tiếp tam giác (ABC).
Hơn nữa ∠N M Y = ∠PQX = ∠X AB = ∠BC Y , suy ra Y thuộc C N ; tương tự Y thuộc
BM , từ đó ta có Y là giao điểm của BM và C N , điều cần chứng minh.

Tập san Toán học STAR EDUCATION


30

Ví dụ 2.20. (China TST 2021). Cho đường tròn w và dây cung U V , tiếp tuyến tại
U, V cắt nhau tại A, gọi O là tâm của (AU V ) và D là trung điểm AU. Đường tròn w0
tâm C qua A, U cắt cung U V tại P và cắt AV tại Q sao cho trung điểm R của cung nhỏ
PQ thuộc đường cao từ U của tam giác AU V . PQ cắt AU tại L, N là trung điểm AL,
M là trung điểm DR, X là hình chiếu của N trên OM .
Chứng minh tứ giác DM T C nội tiếp. „

N
T
A

Q
K
C F
E M
R S D
H
P O

V U

Lời giải. Do ∠P V U = ∠V U P và ∠P V U = ∠PUQ = ∠PQV nên P là điểm V − dumpt y


của tam giác QU V . C P cắt AV tại H, khi đó tam giác UQH cân tại U và R là tâm đường
tròn ngoại tiếp tam giác QE P. Ta có ∠RAU + ∠PUA = ∠QAU + ∠RUA = 90◦ , từ đó
AR⊥U H. R là trực tâm tam giác AU H, do đó HR⊥AU.
Vẽ đường kính AF của w0 thì HAF U là hình bình hành, từ đó C M // H F .
Ta chứng minh U D⊥LT . Thực vậy LH 2 − LD2 + DT 2 − T U 2 = LH 2 − H V 2 − (LD2 −
DU 2 ) = LH 2 −HQ·HA−(LD2 −AD2 ) = LH 2 −H K ·HA−(LA· LU) = LH · LK − LA· LU = 0
Từ đó C M ⊥C N tại tại S.
Kết thúc ta có OS · ON = OM · OT = OC · OD, do đó DM T C nội tiếp.

4. Bài tập rèn luyện


Bài 1. Cho tam giác ABC các đường cao BD, C E, gọi P, Q là chân đường vuông từ D, E
xuống BC. Đường thẳng qua P vuông góc AC và đường thẳng qua Q vuông góc AB cắt
nhau tại M . Chứng minh AM là đường đối trung của tam giác ABC.

Tập san Toán học STAR EDUCATION


31

Bài 2. (APMO 2012) Cho tam giác nhọn ABC. Gọi D là chân đường cao hạ từ A đến
BC, M là trung điểm BC và H là trực tâm của tam giác ABC. E là giao điểm của tia
M H với đường tròn ngoại tiếp tam giác ABC. F là giao điểm của E D với (ABC) khác E.
Chứng minh rằng
BF AB
=
CF AC
Bài 3. (USA TST 2007) Gọi O là đường tròn ngoại tiếp tam giác ABC chot r c. Tiếp tuyến
của O tại các đỉnh B và C gặp nhau tại một điểm T . Xem xétS là giao điểm của đường
thẳng qua A vuông góc với AT với đường biên BC. Biểu thị bằng B1 và C1 các điểm trên
dòng S T , trong đó B1 T = BT = C1 T và sao cho C1 nằm giữa các điểm B1 và S. Khi đó,
các tam giác ABC và AB1 C1 trực tiếp tương tự.
Bài 4. (Đề thi chọn đội tuyển trường PTNK năm 2010) Cho tam giác ABC nội tiếp
đường tròn (O) có A cố định và B, C thay đổi trên (O). Các tiếp tuyến của (O) tại B và
C cắt nhau tại K.Gọi M là trung điểm của BC, N là giao điểm của AM với (O). Chứng
minh đường thẳng K N luôn đi qua một điểm cố định.
Bài 5. Cho tam giác ABC nội tiếp đường tròn (O) và P là điểm sao cho P B, P C là các
tiếp tuyến tuyến của (O). Trên AB, AC ta lấy các điểm K và H sao cho P K k AC, P H k AB.
Chứng minh rằng các điểm H, K và trung điểm các cạnh AB, AC cùng thuộc một đường
tròn.
Bài 6. Cho đường tròn (O) và hai điểm A, B cố định với M là trung điểm AB. Điểm C
thay đổi trên cung lớn AB. Đường trung trực của AC và BC cắt C M tại D và E. Gọi F
là giao điểm của AD và BE. Chứng minh rằng C F luôn đi qua một điểm cố định khi C
thay đổi.
p
Bài 7. Cho tam giác ABC nội tiếp đường tròn (O; R) và BC = R 3. Đường cao AD,
trung điểm M của BC, gọi D0 là điểm đối xứng của D qua M . Tiếp tuyến tại B, C của
(O) cắt nhau tại P. Đường thẳng qua D0 vuông góc với P D0 cắt AB, AC tại F và E. Chứng
minh E, F, P, K cùng thuộc một đường tròn và (KC E) và (K BF ) tiếp xúc nhau với K đối
xứng với A qua M .
Bài 8. Cho tam giác ABC, đường tròn nội tiếp tâm I tiếp xúc với BC, AC, AB tại D, E, F .
J là tâm đường tròn bàng tiếp góc A, P thuộc cung E F không chứa D của (I), Q thuộc
BC sao cho JQ k DP. Chứng minh đường tròn ngoại tiếp tam giác P BC qua trung điểm
của JQ.
Bài 9. (Kì thi hình học Sharigyn năm 2010) Một điểm B thay đổi trên dây AC của
đường tròn w. Đường tròn đường kính AB và BC có tâm là O1 và O2 cắt w lần lượt tại
D và E. Tia O1 D và O2 E cắt nhau tại F , tia AD và C E cắt nhau tại G. Chứng minh F G
qua trung điểm của đoạn AC.
Bài 10. Cho tam giác ABC. Một đường thẳng d thay đổi luôn song song với BC cắt AB
và AC lần lượt tại M , N . Gọi I là giao điểm của BN và C M . Đường tròn ngoại tiếp tam
giác BI M và C I N cắt nhau tại P (khác I). Chứng minh P luôn thuộc một đường thẳng
cố định khi d thay đổi.
Bài 11. Cho tam giác ABC nhọn thỏa điều kiện AB > BC, AC > BC. Gọi H và O lần
lượt là trực tâm và tâm đường tròn ngoại tiếp tam giác ABC. Giả sử đường tròn ngoại
tiếp tam giác AH C cắt đường thẳng AB tại điểm M khác A, và đường tròn ngoại tiếp tam
giác AH C cắt đường thẳng AC tại điểm N khác A. Chứng minh tâm đường tròn ngoại
tiếp tam giác M N H thuộc đường thẳng OH.

Tập san Toán học STAR EDUCATION


32

Bài 12. Cho tam giác nhọn ABC nội tiếp đường tròn tâm O. BE, C F là các đường cao,
H là trưc tâm. M là trung điểm cạnh BC. Tia M H cắt đường tròn ngoại tiếp tam giác
ABC tại D. Các đường thẳng DE, DF cắt lại đường tròn ngoại tiếp tam giác ABC tại P, Q
tương ứng. Chứng minh rằng AO, BQ, C P đồng qui.

Bài 13. Gọi H là trực tâm tam giác ABC, P là điểm nằm trong mặt phẳng tam giác ABC
khác A, B, C. Các điểm L, M , N lần lượt là chân đường vuông góc hạ từ H đến đường
thẳng PA, P B, P C. Gọi X , Y, Z lần lượt là giao điểm của LH, M H, N H với BC, CA, AB.
Chứng minh ba điểm X , Y, Z thẳng hàng.

Bài 14. Cho tam giác ABC cố định, P là điểm thay đổi trên đường cao từ A. Gọi BX , C Y
là đường kính của (PAB), (PAC), cắt nhau tại Q. Chứng minh rằng Q di chuyển trên
một đường thẳng cố định.

Bài 15. Cho tam giác ABC, các đường cao AA1 , BB1 , C C1 cắt nhau tại H. Đường thẳng
qua H vuông góc với B1 C1 , A1 C1 cắt các tia CA, C B tại P, Q tương ứng. Chứng minh rằng
đường thẳng qua C vuông góc với A1 B1 qua trung điểm của PQ.

Bài 16. Cho tam giác ABC, gọi O là tâm đường tròn ngoại tiếp tam giác. Gọi A0 là hình
chiếu của A trên BC, gọi X là điểm thuộc tia AA0 . Đường phân giác trong góc BAC cắt
đường tròn ngoại tiếp tại D. Gọi M là trung điểm của đoạn DX . Đường thẳng qua O
song song với AD cắt đường thẳng DX tại N . Chứng minh rằng ∠BAM = ∠CAN .

Bài 17. Cho tam giác ABC có O là tâm đường tròn ngoại tiếp, P là một điểm thay đổi
trên đường cao AD của tam giác ABC. Gọi I, J lần lượt là tâm đường tròn ngoại tiếp
các tam giác AP B, AP C. Chứng minh tâm của đường tròn ngoại tiếp tam giác OI J thuộc
một đường cố định.

Bài 18. Cho tam giác ABC có D là tâm đường tròn nội tiếp. Hai điểm P, Q thuộc AI sao
cho ∠P BA = ∠P BC, ∠QCA = ∠QC B. Các điểm M , N thuộc đường thẳng BC sao cho
MB NB
= . Gọi H là hình chiếu của M trên P N . Chứng minh ∠QM C = ∠AH P.
MC NC

Bài 19. Cho tam giác ABC, D là điểm thuộc đường đối trung sao cho ∠BDC = 2∠BAC,
cùng phía với A.E là điểm thuộc cạnh BC không thuộc AD. Đường tròn w có tâm nằm
trên đường thẳng vuông góc với AD tại D và đi qua A, E, w cắt (BDC) tại F sao cho E, F
khác phía với AD. Chứng minh AD cũng là đường đối trung của tam giác AE F .

Bài 20. (Nga 2010) Cho tam giác ABC nội tiếp đường tròn (O), M là điểm nằm
trong tam giác sao cho M B = M C. Gọi I1 , I2 là tâm đường tròn nội tiếp các tam giác
ABM , AC M .

1. Chứng minh (AI1 I2 ) qua điểm chính giữa cung BC, gọi là D.

2. Gọi E là điểm đối xứng của D qua O. Chứng minh ∠E I1 B = ∠E I2 C.

Bài 21. Cho tam giác ABC nhọn có H là trực tâm. Điểm P di chuyển bên trong tam giác
sao cho P thuộc đường tròn ngoại tiếp tam giác (H BC), đường thẳng đối xứng với BP
qua phân giác góc B và đường thẳng đối xứng với C P qua phân giác góc C cắt nhau ở
Q. Đường trung trực AP cắt các đường thẳng AB, AC lần lượt tại F, E. Chứng minh QA
là phân giác góc FQE.

Tập san Toán học STAR EDUCATION


33

Bài 22. Cho tam giác ABC nhọn, P là điểm nằm trong tam giác sao cho ∠PAC = ∠P C B.
Gọi X , Y, Z lần lượt là hình chiếu của P trên BC, AC, AB. Gọi O là tâm đường tròn ngoại
tiếp tam giác X Y Z. H là chân đường cao vẽ từ B đến AC, N là trung điểm AC. Lấy điểm
T sao cho T Y PO là hình bình hành. Chứng minh hai tam giác T Y N và P BC đồng dạng.

Bài 23. Cho tam giác ABC nhọn nội tiếp đường tròn tâm O, đường cao AD cắt (O) tại
K. Các điểm P, Q thay đổi trên AD và AO sao cho P, Q thuộc miền trong tam giác và
∠P BA = ∠QBC. Vẽ QT ⊥BC. Gọi K 0 đối xứng với K qua O, K 0 T cắt (O) tại L. Gọi E, F
là hình chiếu của P trên AB, AC. Chứng minh giao điểm của LK và E F thuộc một đường
cố định.

Bài 24. Gọi P1 , P2 là các liên từ đẳng giác với tam giác ABC. Điểm Q 1 nằm trên đường
tròn ngoại tiếp tam giác BC P1 sao cho các điểm P1 và Q 1 đối nhau theo đường kính và
Q 2 được xây dựng tương tự. Chứng minh rằng Q 1 , Q 2 cũng là các liên từ đẳng giác đối
với tam giác ABC

Bài 25. Gọi ABC là hình tam giác và Γ là hình tròn cắt đường thẳng BC tại A1 , A2 ,
đường thẳng CA tại B1 và B2 và dòng C1 và C2 . Gọi Ω1 , Ω2 , Ω3 là các đường tròn có
đường kính A1 A2 , B1 B2 và C1 C2 , tương ứng. Chứng minh rằng tâm gốc của ba đường
tròn này là liên hợp đẳng giác của tâm Γ đối với tam giác ABC.

Bài 26. (USAMO 2008) Gọi ABC là tam giác nhọn, vô hướng và đặt M , N và P lần lượt
là trung điểm của các đoạn BC, CA và AB. Cho các đường trung trực của các đoạn AB và
AC cắt tia AM lần lượt tại các điểm D và E, và cho các đường thẳng BD và C E cắt nhau
tại điểm F , bên trong tam giác ABC. Chứng minh rằng các điểm A, N , F và P cùng nằm
trên một đường tròn.

Bài 27. (ELMO 2014) Cho ABC D là tứ giác nội tiếp trong đường tròn ω. Xác định
E = AA ∩ C D, F = AA ∩ BC, G = BE ∩ ω, H = BE ∩ AD. I = DF ∩ ω, và J = DF ∩ AB.
Chứng minh rằng GI, H J cắt nhau tại một điểm nằm trên đường đối trung đỉnh B của
tam giác ABC đồng quy.

Bài 28. Cho tam giác ABC nhọn và có một điểm P nằm trong tam giác sao cho ∠AP B =
∠AP C = α và ∠α > 180o − ∠BAC. Đường tròn ngoại tiếp tam giác AP B cắt AC tại E;
đường tròn ngoại tiếp tam giác AP C cắt AB tại F . Gọi Q là điểm nằm trong tam giác
AE F sao cho ∠AQE = ∠AQF = α. Gọi D là điểm đối xứng của Q qua đường thẳng E F .
Phân giác góc ∠E DF cắt AP tại điểm T .

1. Chứng minh rằng ∠DE T = ∠ABC và ∠AF T = ∠AC B.

2. Đường thẳng AP cắt các đường thẳng DE và DF tại M , N . Gọi I là tâm đường
tròn nội tiếp tam giác E P M và J là tâm đường tròn nội tiếp tam giác F P N . Gọi K
là tâm đường tròn ngoại tiếp tam giác DI J. DT cắt (K) tại H. Chứng minh đường
thẳng K H đi qua tâm đường tròn nội tiếp tam giác DM N .

Tập san Toán học STAR EDUCATION


Tài liệu tham khảo

[1] Hoàng Chúng, 1996, Hình học của tam giác NXB Giáo dục.

[2] Trần Văn Tấn (chủ biên), 2005, Các chuyên đề hình học bồi dưỡng học sinh giỏi
Trung học cở sở NXB Giáo dục.

[3] Đỗ Thanh Sơn, 2010, Một số chuyên đề hình học phẳng bồi dưỡng học sinh giỏi
THPT NXB Giáo dục.

[4] Roger A.Johnson, 1964, Advanced Euclidean Geometry Dover Publications.

[5] Titu Adreescu và Cosmin Pohoata, 2016, Lemmas in Olympiad Geometry. XYZ
press.

[6] Evan Chen, 2015, Euclidean Geometry mathematical Olympiads. MAA press,
2015.

[7] Trần Quang Hùng, 2017, Mỗi tuần một bài toán Hình học.

[8] Nguyễn Văn Linh, 2018, 108 Bài toán hình học sơ cấp, NXB ĐHQG HN,2018.

[9] TẠP CHÍ TOÁN HỌC VÀ TUỔI TRẺ, 2013, 2014, 2015, 2016, 2017.

34
Tuyển tập số học thi HSG cấp tỉnh
năm 2021 - 2022

Lê Phúc Lữ (ĐH Khoa học tự nhiên TPHCM)


Nguyễn Công Thành (Fulbright University Vietnam, TPHCM)

(∗) Tài liệu có tham khảo từ đáp án đề thi chọn đội tuyển các tỉnh.

1. Các bài toán chọn lọc

Bài toán 1 (Hà Nam). Với X là một tập hợp các số thực, ta kí hiệu S (X ) là tổng các
phần tử thuộc tập X . Một tập A gồm các số nguyên dương được gọi là tập “nguyên tố”
nếu với mọi tập con B khác rỗng của tập A thì gcd (S(B), S(A)) = 1.

1. Tìm một tập “nguyên tố” gồm 6 phần tử.

2. Tìm tất cả các số nguyên dương n để tồn tại a, b ∈ N sao cho tập

A = (a + b)2 , (a + 2b)2 , ..., (a + nb)2




là tập “nguyên tố”.

Lời giải.
1) Xét tập A = {4; 9; 16; 25; 36; 49} ta có S (A) = 139 là số nguyên tố. Do đó với mọi
B ⊂ A thì gcd (S(A), S(B)) = 1. Suy ra A là tập “nguyên tố”.
2) Nếu n |6 thì ta chỉ ra được tập A là tập “nguyên tố” với a = b = 1.
Xét n không là ước của 6. Ta có
n
X n
X
2

S (A) = (a + i b) = a2 + 2abi + b2 i 2
i=1 i=1
n n
X X n(n + 1)(2n + 1)
= na + 2ab
2
i+b 2
i 2 = na2 + abn (n + 1) + b2
i=1 i=1
6
b (n + 1) (2n + 1)
2
• ˜
= n a + ab (n + 1) +
2
.
6

Suy ra tồn tại ước nguyên tố p của n sao cho p | S(A). Ta xét các trường hợp sau đây:

35
36

• Nếu b ≡ 0 (mod p) thì x ≡ a2 (mod p), ∀x ∈ A. Suy ra


p
X
(a + i b)2 ≡ a2 p ≡ 0 (mod p)
i=1

nên B = (a + b)2 , (a + 2b)2 , ..., (a + pb)2 ⊂ A thỏa mãn gcd (S(A), (SB)) > 1.


• Xét gcd (b, p) = 1, khi


 đó tồn tại i ≤ n để bi ≡ −a (mod p) nên a + bi ≡ 0
(mod p). Xét tập B = (a + i b)2 ⊂ A và gcd (S(A), S(B)) > 1.

Vậy n ∈ {1, 2, 3, 6} là tất cả những giá trị cần tìm.

n3n−2 −3n+1
Bài toán 2 (Hà Tĩnh). Tìm tất cả các số nguyên dương n sao cho 3n−2 là một số
nguyên.

Lời giải. Giả sử tồn tại số nguyên dương n thỏa mãn bài toán. Khi đó

3n − 2 | n3n−2 − 3n + 1 ⇒ n3n−2 ≡ 1 (mod 3n − 2)

Nếu n chẵn thì 3n − 2 chẵn và n3n−2 − 3n + 1 lẻ nên không thỏa mãn, do đó phải có n
lẻ, suy ra 3n − 2 lẻ.

• Kiểm tra được n = 1 thỏa mãn.

• Xét n 6= 1 thì n ≥ 3 nên 3n − 2 > 3.

Gọi p là ước nguyên tố nhỏ nhất của 3n − 2, ta có p lẻ và (p , n) = 1. Khi đó


theo Định lí Fermat nhỏ ta có n p−1 ≡ 1 (mod p) và từ đồng dư đầu tiên, ta cũng
có n3n−2 ≡ 1 (mod p).

Đặt r = ord p (n). Khi đó r | p − 1 và r | 3n − 2 nên r | (p − 1 ; 3n − 2). Suy ra r


là một ước nguyên dương của 3n − 2 và nhỏ hơn p. Do đó, nếu r 6= 1 thì r có
ước nguyên tố q nào đó, dẫn tới q là ước nguyên tố của 3n − 2 và q ≤ r < p,
điều này mâu thuẫn với tính nhỏ nhất của p.

Vậy phải có r = 1, tức là n ≡ 1 (mod p) nên p | n−1. Mà p | 3n−2 = 3 (n − 1)+


1 nên p | 1, vô lí.

Vậy chỉ có duy nhất n = 1 thỏa mãn bài toán.

Bài toán 3 (Hải Phòng). Cho p là một số nguyên tố lẻ. Xác định số nguyên dương n
thoả mãn n + p2021 là một luỹ thừa của p và p2022 | C2n
n

Lời giải. Đặt n + p2021 = p m (m ∈ N; m > 2021) , ta có n = p m − p2021 .


Ta có 
n
vp C2n = vp ((2n)!) − 2vp (n!) ,
+∞ • ˜ 2021
X• n ˜ m • ˜
X n X n
vp (n!) = k
= k
+ k
,
k=1
p k=1
p k=2022
p

Tập san Toán học STAR EDUCATION


37

+∞ • 2021 m
2n 2n 2n
X ˜ X• ˜ X • ˜
vp ((2n)!) = = + .
k=1
pk k=1
pk k=2022
p k

Suy ra
m •
2n
˜ • ˜‹
 X n
n
vp C2n = − 2. k .
k=1
pk p

Chú ý rằng
• ˜ 
2n 0 ∀k = 1; 2021
• ˜
n
− 2. =
p k p k 1 ∀k = 2022; m

n
nên vp C2n = m − 2021. Từ đó

n .. 2022
C2n .p ⇔ m − 2021 ≥ 2022 ⇔ m ≥ 4043.

Vậy n = p m − p2021 , m ∈ N; m ≥ 4043.

Bài toán 4 (Tp Hồ Chí Minh). Giả sử n là số nguyên dương lớn hơn 1 thỏa mãn n
và 2022n − 1 + (n − ϕ(n))! có cùng tập ước nguyên tố (trong đó ϕ là hàm Euler).

a) Chứng minh rằng n là hợp số và (n, 6) = 1.

b) Chứng minh rằng (n, 42!) = 1.

Lời giải.
a) Đặt k = n − ϕ(n). Nếu n là số nguyên tố thì ϕ(n) = n − 1 nên k = 1. Từ đó suy ra n
và 2022n có cùng tập ước nguyên tố. Suy ra n chia hết cho 2022, vô lý. Do đó, n phải
là hợp số.
Vậy k = n − ϕ(n) ≥ 2 nên k! chẵn, kéo theo 2022n − 1 + k! lẻ và n cũng lẻ. Giả sử 3 | n
thì để ý 3 | 2022n nên 3 | (−1 + k!), vô lý do k ≥ 2. Do đó 3 - n và dẫn đến (n, 6) = 1.
b) Gọi p là ước nguyên tố nhỏ nhất của n thì n ≥ p2 . Khi đó các số p, 2p, . . . , n không
nguyên tố cùng nhau với n nên k = n − ϕ(n) ≥ np ≥ p. Suy ra k! chia hết cho p. Lại có
2022n − 1 + k! chia hết cho p nên p | 2022n − 1.
Suy ra (p, 2022) = 1. Từ đó, theo định lý Fermat nhỏ thì p | 2022 p−1 − 1. Gọi h là cấp
của 2022 modulo p thì h | p − 1 và h | n nên h | (p − 1, n) = 1, điều này có được do p là
ước nguyên tố nhỏ nhất của n. Từ đó h = 1 nên p | 2022 − 1 = 2021 hay p ∈ {43, 47}.
Suy ra (n, 42!) = 1.

Bài toán 5 (Huế). Có tồn tại hay không 2022 số nguyên tố phân biệt p1 , p2 , ..., p2022
2022
P 1
và số nguyên dương n thỏa mãn p2 +1
= n12 ?
i
i=1

Lời giải. Giả sử tồn tại 2022 số nguyên tố phân biệt p1 , p2 , ..., p2022 và số nguyên
dương n thỏa mãn
2022
X 1 1
= 2.
p +1 n
i=1 i
2

Tập san Toán học STAR EDUCATION


38

Đặt
2022
‚2022 Œ
Y X P
P= (pi 2 + 1) ⇒ n2 =P (*)
i=1 i=1
pi + 1
2

• Nếu pi 6= 3, ∀i = 1; 2; ...; 2022 thì theo định lý Fermat,


pi 2 + 1 ≡ −1 (mod 3), ∀i = 1; 2; ...; 2022 ⇒ P ≡ 1 (mod 3)
2022
P
≡ −1 (mod 3), ∀i = 1; 2; ...; 2022, kéo theo P
≡ 0 (mod 3).
P
Do đó pi 2 +1 pi 2 +1
i=1
Điều này mâu thuẫn vì vế trái của (*) chia hết cho 3 và vế phải của (*) đồng dư
với 1 (mod 3).
• Nếu tồn tại một số j ∈ {1; 2; ...; 2022} sao cho và pi 6= 3, ∀i = 1; 2; ...; 2022; i 6= j
Khi đó pi 2 + 1 ≡ ±1 (mod 3), ∀i = 1; 2; ...; 2022; i 6= j. Suy ra P ≡ −1 (mod 3).
2022
P
có một số hạng đồng dư với −1 (mod 3), còn 2021
P
Do đó trong tổng pi 2 +1
i=1
số hạng còn lại đồng dư với 1 (mod 3). Do đó
2022
X P
≡ 1 (mod 3).
i=1
pi 2 + 1

Mặt khác vì n ≡ 0 hoặc 1 (mod 3) nên suy ra


2

‚2022 Œ
X P
n2 ≡ 1 (mod 3)
p 2+1
i=1 i

hoặc ‚2022 Œ
X P
n2 ≡ 0 (mod 3).
i=1
pi + 1
2

Điều này cũng mâu thuẫn vì vế trái của (*) đồng dư với 0 hoặc 1 (mod 3) còn
vế phải của (*) đồng dư với −1 (mod 3).

Bài toán 6 (Nam Định). Cho a và n là các số nguyên dương, a ¾ 2. Chứng minh rằng
n n
mọi ước nguyên tố của a2·6 − a6 + 1 đều có dạng 6n+1 k + 1 với k là số nguyên dương.
n n
Lời giải. Gọi p là một ước nguyên tố của a2·6 − a6 + 1, có
n n
a2·6 − a6 + 1 ≡ 0 (mod p) (1)
n n n n 
Từ a2·6 − a6 + 1 = a6 a6 − 1 + 1 là số lẻ nên p lẻ.
n n n
Vì n ∈ N∗ ⇒ 6n là số chẵn ⇒ a6 chia 3 dư 0 hoặc 1 ⇒ a2·6 − a6 + 1 không chia hết
cho 3 ⇒ p khác 3.
2 n  n n  2
Ta có a3·6 + 1 = a6 + 1 a2·6 − a6 + 1 nên từ (1) suy ra a3·6 + 1 ≡ 0 (mod p), suy
n n 2 n+1
ra a3·6 ≡ −1 (mod p), nên a3·6 ≡ 1 (mod p), hay a6 ≡ 1 (mod p).
Gọi h là cấp của a theo mod p. Ta có h | 6n+1 nên h có dạng h = 3k · 2 t (k, t ≤ n + 1).
xảy ra các trường hợp:

Tập san Toán học STAR EDUCATION


39

.
• Nếu t ≤ n thì 3·6n = 3n+1 ·2n .. h, suy ra a3·6 ≡ 1 (mod p). Theo trên a3·6 +1 ≡ 0
n 2

.
(mod p) nên suy ra 2 .. p ⇒ p = 2, mâu thuẫn với p lẻ.
.
• Nếu t = n + 1 và k ≤ n thì 2 · 6n = 3n · 2n+1 .. h, suy ra a2·6 ≡ 1 (mod p). Kết hợp
n

n n n
với (1) suy ra a6 ≡ 2 (mod p), nên a3·6 ≡ 8 (mod p). Theo trên có a3·6 ≡ −1
(mod p), kéo theo p | 9 ⇒ p = 3, mâu thuẫn.
n n 
• Nếu t = n + 1, k = n + 1 thì h = 6n+1 , do p | a2·6 − a6 + 1 nên (a, p) = 1,
.
áp dụng định lý Fermat ta có a p−1 ≡ 1 (mod p), suy ra p − 1 .. h. Vậy tồn tại k
nguyên dương mà p − 1 = kh hay p = 6n+1 · k + 1, có điều cần chứng minh.

Bài toán 7 (Phú Thọ). Cho a, b, m, n là các số nguyên dương và gcd(a, b) = 1. Chứng
minh rằng
gcd (a m − b m , a n − b n ) = agcd(m,n) − bgcd(m,n) .

Lời giải.

• Trường hợp gcd(m, n) = 1. Ta cần chứng minh gcd (a m − b m , a n − b n ) = a − b.

Dễ thấy (a−b) | gcd (a m − b m , a n − b n ). Ta chứng minh d = gcd (a m − b m , a n − b n ) |


(a−b). Thật vậy, do gcd(m, n) = 1 nên theo định lý Bezout, tồn tại các số nguyên
dương x, y sao cho mx − n y = 1. Ta có

d | am − bm d | a mx − b mx d | a v+1 − b vv+1 d | a v+1 − b vv+1


§ § § §
n n ⇒ vv vv ⇒ v vv ⇒
d|a −b d|a −b d|a −b d | a v b − b vv+1

Suy ra d | a0w (a − b) (∗)

Vì gcd(a, b) = 1, nên gcd (a nv , b) = 1. Mà d | a n − b n , suy ra gcd (d, a nv ) = 1. Từ


(∗) suy ra d | (a − b).

• Trường hợp gcd(m, n) = d. Ta có m = du, n = d v với gcd(u, v) = 1. Khi đó, theo


phần a) thì
u u v v 
gcd a d − b d , a d − b d = a d − b d = agcd(m,n) − bgcd(m,n) .

Bài toán 8 (Quảng Ninh). Cho trước số nguyên dương a và số nguyên tố p.

a) Chứng minh tồn tại vô số số nguyên dương n để p | a n + n.

b Chứng minh với mọi số nguyên dương m, đều tồn tại số nguyên dương n để
p m | a n = n.

Lời giải.
a)

Tập san Toán học STAR EDUCATION


40

• Nếu p | a, ta thấy các n = kp, trong đó k là số nguyên dương tuỳ ý thoả mãn đề
bài.

• Nếu p - a thì a p−1 ≡ 1 (mod p) nên ta chọn n = kp (p − 1) + (p − 1), với k là số


nguyên dương tuỳ . Khi đó

a n + n ≡ a kp(p−1) + kp(p − 1) + p − 1 ≡ 1 + p − 1 ≡ 0 (mod p).

Từ đó suy ra các số n đã chọn thoả mãn yêu cầu.

b)

• Nếu p | a thì chọn n = p m thoả mãn đề bài.

• Nếu p - a, ta chứng minh quy nạp theo m.

Với m = 1, theo phần a), tồn tại số n1 thoả mãn đề bài. Giả sử khẳng định đúng
với m, tức là tồn tại số tự nhiên nm để p m | am
n
+ nm , ta cần chỉ ra tồn tại số nm+1
thoả mãn đề bài.

n
Đặt am + nm = kp m , xét số n = nm + tϕ p m+1 thì

a n + n ≡ a nm + nm + tφ(p m+1 ) ≡ kp m + t p m (p − 1) (mod p m+1 )

nên ta chỉ cần chọn t = t 0 sao cho p | k + t 0 (p − 1) (luôn chọn được vì p và


p − 1 nguyên tố cùng nhau). Khi đó, với nm+1 = nm + t 0 ϕ(p m+1 ) thì chỉ ra được
p m+1 | am+1
n
+ nm+1 . Theo nguyên lí quy nạp, ta có điều cần chứng minh.

Bài toán 9 (Sơn La). Chứng minh rằng nếu số tự nhiên m có dạng 4k + 1 với k là số
nguyên dương, mà biểu diễn được không ít hơn hai cách dưới dạng tổng hai số chính
phương thì m là hợp số.

Lời giải. Giả sử số m có dạng 4k + 1 với k là số nguyên dương, có hai các biểu diễn
dưới dạng tổng bình phương của hai số

m = x 2 + y 2 = u2 + v 2 .

Vì m là số lẻ nên trong mỗi tổng phải có một số lẻ. Không mất tính tổng quát, giả sử
x và u là số lẻ, còn y và v là số chẵn, và x > u, ta có y < v, và các số x ± u và v ± y
là các số nguyên dương chẵn. Ta viết lại m dưới dạng
x +u x − u 2  v + y v − y 2
m= + + −
2 2 2 2
 x + u 2 x − u 2  v + y 2  v − y 2
= + + +
2 2 2 2
= p +q +r +s ,
2 2 2 2

v+ y v− y
trong đó x+u
2 = p, x−u
2 = q, 2 = r, 2 = s.

Tập san Toán học STAR EDUCATION


41

Gọi a là ước chung lớn nhất của p và r. Giả sử p = ab, r = ac, trong đó gcd (b, c) = 1,
khi đó ta có abq = acs nên bq = cs, kéo theo c | q.
Giả sử q = cd, ta có
bcd = cs
 
nên s = bd, và như vậy m = a2 b2 + c 2 d 2 + a2 c 2 + b2 d 2 = a2 + d 2 b2 + c 2 , tức m là
hợp số, vô lý. Vậy điều đã giả sử là sai, hay ta có điều cần chứng minh.

Bài toán 10 (Yên Bái). Tìm tất cả các số nguyên tố p sao cho tồn tại các số nguyên
dương n, x, y thoả mãn

pn = x 3 + y 3 (*)

Lời giải. Với p = 2 và p = 3, dễ thấy (n, x, y) = (1, 1, 1) và (n, x, y) = (2, 2, 2) thoả


mãn. Ta chứng minh khi p > 3, không tồn tại n, x, y thoả mãn yêu cầu đề bài.
Thật vậy, giả sử ngược lại, ta chọn n, x, y thoả mãn (*) sao cho n bé nhất. Do p 6= 2
nên (x, y) 6= (1, 1). Khi đó

x 2 − x y + y 2 = (x − y)2 + x y > 1 và x + y > 1.

Do đó x 2 − x y + y 2 và x + y đều là bội của p. Suy ra

.
(x + y)2 − x 2 − x y + y 2 = 3x y .. p.


Do p > 3 nên p | x hoặc p | y. Mà p | x + y nên 3 | x, 3 | y. Điều này cho ta


 ‹3  ‹3
x y
(*) ⇔ p n−3
= + ,
p p
€ Š
y
hay nói cách khác, bộ số n − 3, xp , p cũng thoã mãn đề bài, điều này mâu thuẫn với
tính bé nhất của n.
Vậy chỉ có p = 2 và p = 3 thoã mãn yêu cầu bài toán.

Bài toán 11 (Trải nghiệm VMO 2022, vòng 1). Một số nguyên dương được gọi là
gần chính phương nếu nó là tích của hai số nguyên dương liên tiếp. Một số nguyên
dương được gọi là biểu diễn được nếu nó biểu diễn được thành thương của hai số gần
chính phương.

a) Chứng minh mọi số gần chính phương và mọi số nguyên dương có dạng 4(2k +
1)2 (với k là số nguyên dương) đều là số biểu diễn được.

b) Cho p là số nguyên tố và s là số nguyên dương. Chứng minh mọi số nguyên


dương có dạng p2s không là số biểu diễn được.

Tập san Toán học STAR EDUCATION


42

Lời giải.
a) Với mọi số nguyên dương n và k, ta có:

n(n + 2)(n + 1)2 (n2 + 2n)(n2 + 2n + 1)


n(n + 1) = =
(n + 1)(n + 2) (n + 1)(n + 2)


4k(k + 1)(2k + 1)2 (4k2 + 4k)(4k2 + 4k + 1)
4(2k + 1)2 = =
k(k + 1) k(k + 1)
nên mọi số gần chính phương và mọi số nguyên dương có dạng 4(2k + 1)2 đều là số
biểu diễn được.
b) Giả sử tồn tại số nguyên tố p và số nguyên dương s sao cho p2s > 1 là số biểu diễn
m(m+1)
được. Khi đó tồn tại các số nguyên dương m, n (m > n) sao cho p2s = n(n+1) hay

n(n + 1)p2s = m(m + 1). (1)

Từ đó suy ra p2s |m(m + 1) nên ta xét các trường hợp:


Trường hợp 1. p2s |m. Khi đó tồn tại số nguyên dương k sao cho m = kp2s , thay vào
(1) ta có n(n+1) = k(kp2s +1), từ đó suy ra (2n+1)2 = 4n(n+1)+1 = 4(kps )2 +4k+1
là số chính phương. Mặt khác ta lại có:

(2kps )2 = 4(kps )2 < 4(kps )2 + 4k + 1 < 4(kps )2 + 4kps + 1 = (2kps + 1)2 .

Do (2kps )2 và (2kps + 1)2 là hai số chính phương nên ta có điều mâu thuẫn.
Trường hợp 2. p2s |m + 1. Khi đó tồn tại số nguyên dương k sao cho m + 1 = kp2s ,
thay vào (1) ta có n(n + 1) = (kp2s − 1)k, từ đó suy ra (2n + 1)2 = 4k(kp2s − 1) + 1 =
4(kps )2 − 4k + 1 là số chính phương. Mặt khác ta lại có:

(2kps − 1)2 = 4(kps )2 − 4kps + 1 < 4(kps )2 − 4k + 1 < 4(kps )2 = (2kps )2 .

Do (2kps )2 và (2kps + 1)2 là hai số chính phương nên ta cũng có điều mâu thuẫn. Vậy
không tồn tại số biểu diễn được có dạng p2s với p nguyên tố và s nguyên dương.

m
Bài toán 12 (Trải nghiệm VMO 2022, vòng 2). Cho Am = 11 +33 +· · ·+(2m − 1)2 −1
với mọi số nguyên dương m lớn hơn 1. Chứng minh rằng Am chia hết cho 2m nhưng
không chia hết cho 2m+1 .

Lời giải. Để giải quyết bài toán trước hết ta sẽ đi chứng minh bổ đề quen thuộc sau
Bổ đề. Cho n là số nguyên dương. Chứng minh rằng với mọi số nguyên dương k lẻ
n
thì (2n + k)k ≡ k k (2n + 1) (mod 2n+2 ) và k2 ≡ 1 (mod 2n+2 ).
Chứng minh. Thật vậy, ta có các đẳng thức sau

(2n + k)k = 2nk + · · · + Ck1 · 2n · k k−1 + k k


2n
 € 2n−1 Š
k − 1 = (k − 1)(k + 1) k + 1 · · · k
2
+1

Tập san Toán học STAR EDUCATION


43

n 
Để ý rằng 2n ¾ n + 2 với n > 1 và v2 k2 + 1 = 1 nên

(2n + k)k ≡ k · 2n · k k−1 + k k ≡ k k (2n + 1) (mod 2n+2 )


n 
v2 k2 − 1 = v2 (k − 1) + v2 (k + 1) + n − 1 ≥ n + 2

Trờ lại với bài toán, áp dụng bổ đề trên thì


n
+1 n+1
An+1 = An + (2n + 1)2 + · · · + (2n+1 − 1)2 −1

n
≡ An + (2n + 1)1 + · · · + (2n+1 − 1)2 −1
(mod 2n+2 )
≡ An + (2n + 1)An ≡ (2n + 2)An (mod 2n+2 )

Từ đây bằng quy nạp ta dễ dàng có được điều phải chứng minh.

Bài toán 13 (Trải nghiệm VMO 2022, vòng 3). Một tập S gồm các số nguyên dương
được gọi là đẹp nếu S có ít nhất 2 phần tử và ∀a, b ∈ S mà a 6= b thì a − b là ước của
a và b nhưng không là ước của bất cứ phần tử nào khác của S. Chứng minh rằng với
mọi n nguyên dương, n ≥ 2, tồn tại một tập S đẹp gồm n phần tử.

Lời giải. Ta chứng minh tồn tại 1 tập đẹp gồm n phần tử bằng quy nạp theo n ¾ 2.
Với n = 2, rõ ràng tập {1, 2} là đẹp.
Giả sử tồn tại tập S = {a1 , a2 , · · · , ak } là đẹp. Đặt A = [a1 , a2 , · · · , ak ].
Nhận xét. S 0 = {m(a1 + tA), m(a2 + tA), · · · , m(ak + tA)} cũng là tập đẹp với mọi
m, t ∈ N∗ .
Chứng minh. Thật vậy: m(ai + tA) − m(a j + tA) = m(ai − a j ) là ước của mai , ma j .
Mà mai |mA nên m(ai − a j ) là ước của m(ai + tA) và m(a j + tA). Hơn nữa nếu m(ai −
a j )|m(ak + tA) với k khác i, j thì m(ai − a j )|mak (mâu thuẫn). Suy ra nhận xét trên là
đúng.
Ta sẽ chọn t,m và 1 phần tử y thêm vào S 0 để thu được 1 tập đẹp gồm k + 1 phần tử.
Ta sẽ chọn y có dạng y = x + mtA. Như vậy, ta cần chọn t, m, x thỏa mãn các điều
kiện:

1. x − mai |m(ai + tA) , ∀1 ≤ i ≤ k.

2. m(ai − a j ) - x + mtA ; ∀1 ≤ i, j ≤ k.

3. x − mai - m(a j + tA).

Mà ai − a j |A nên điều kiện thứ hai tương đương với m(ai − a j ) - x.


Như vậy ta chỉ cần chọn x nguyên tố cùng nhau với mA thì x nguyên tố cùng nhau
với m(ai − a j ) và thoả mãn điều kiện thứ hai. Ta chọn x = mA + 1.
Ta sẽ dùng định lí thặng dư Trung Hoa để chọn t thỏa (1). Như vậy, ta cần x −ma1 , x −
ma2 , · · · , x − mak đôi 1 nguyên tố cùng nhau và từng số x − mai nguyên tố cùng nhau
với mA.
Để ý rằng (x − mai , x − ma j ) = (x − mai , m(ai − a j ) = (x, m(ai − a j )) = 1 , ∀i 6= j.
Như vậy ta chỉ cần chọn x để (x − mai , mA) = 1 , ∀1 ≤ i ≤ k.
Ta lại có (x − mai , mA) = (mA + 1 − mai , mA) = (1 − mai , mA).
Nếu ta chọn m = A thì (1 − mai , mA) = (1 − mai , m2 ) = 1. (do (1 − mai , m) = 1).

Tập san Toán học STAR EDUCATION


44

Tóm lại ta chọn m = A và x = mA + 1 = A2 + 1. thì sẽ tồn tại t thỏa mãn:

mAt + mai ≡ 0 (mod x − mai ) , ∀1 ≤ i ≤ k.

Như vậy bộ (m, t, x) vừa chọn sẽ thỏa mãn điều kiện 1 và 2.


Ta chứng minh bộ (m, t, x) vừa chọn cũng thỏa mãn điều kiện 3.
Thật vậy: (x − mai , m(ai − a j )) = (x, m(ai − a j )) = 1. Vậy nên x − mai - m(ai − a j ), mà
x − mai |m(ai + tA) nên x − mai - m(a j + tA)(thỏa mãn điều kiện 3).
Vậy S ∗ = {m(a1 + tA), m(a2 + tA), · · · , m(ak + tA), y} là tập đẹp gồm k + 1 phần tử với
m, t, x đã chọn như trên và y = x + mtA.
Theo nguyên lí quy nạp thì với mọi n nguyên dương, n ≥ 2 tồn tại tập đẹp gồm n
phần tử.

Bài toán 14 (Ninh Bình).

xy
a) Cho số nguyên dương d, tìm cặp số (x, y) nguyên dương sao cho x− y = d.

b) Với mỗi số nguyên dương n > 1, gọi f (n) là số cặp (x, y) nguyên dương sao cho
xy
x− y là ước nguyên dương của n. Tìm tất cả các giá trị của n để f (n) = 231.

Lời giải.
xy
a) Với số nguyên dương d, ta có x− y = d thì (x + d)( y − d) = −d 2 . Khi đó, tồn tại
d2
ước nguyên dương r của d 2 sao cho r = x + d, thế thì x = r − d và y = d − r . Do
x > y > 0 nên r > d.
Ngược
€ lại, vớiŠmỗi ước nguyên dương r > d của d 2 , thử trực tiếp, ta được cặp (x, y) =
2 xy
r − d, d − dr thoả mãn x− y = d.

xy τ(d 2 )−1
Như vậy số cặp (x, y) nguyên dương thoả x− y = d chính là 2 .
b) Rõ ràng !

X τ d2 − 1 1X  τ(n)
f (n) = = τ d2 − .
d|n
2 2 d|n 2
s
Y
a

Dễ thấy τ m 2
là một hàm nhân tính. Giả sử n có phân tích tiêu chuẩn là n = pi i ,
i=1
sử dụng công thức tổng trải, ta được

X s
Y
2a
   
τ d 2
= 1 + τ pi2 + τ pi4 + · · · + τ pi i
d|n i=1
s
Y
= (1 + 3 + 5 + · · · + (2ai + 1))
i=1
Ys
= (1 + ai )2 = (τ(n))2 .
i=1

Tập san Toán học STAR EDUCATION


45

Điều này dẫn tới


(τ(n))2 − τ(n) τ(n)(τ(n) − 1)
f (n) = = ,
2 2
do đó f (n) = 231 khi τ(n)(τ(n) − 1) = 22 · 21. Suy ra
s
Y
(1 + ai ) = τ(n) = 22 = 2 · 11.
i=1

Vậy hoặc n có đúng một ước nguyên tố p và n = p21 hoặc n có đúng hai ước nguyên
tố phân biệt p, q và n = pq10 .

Bài toán 15 (Vĩnh Phúc). Cho n ¾ 2 số tự nhiên a1 < a2 < · · · < an sao cho an < 2a1 .
Gọi m là số các ước nguyên tố phân biệt của a1 a2 . . . an . Chứng minh rằng

(a1 a2 . . . an )m−1 ¾ (n!)m .

Lời giải. Gọi p là một ước số nguyên tố của tích a1 a2 . . . an . Ta đặt ai = p ki bi , trong đó
bi không chia hết cho p (i = 1, 2, . . . , n). Tiếp theo, vì a1 < a2 < · · · < an < 2a1 , chúng
ta thấy rằng tất cả các bi là đôi một phân biệt. Thật vậy, nếu bi = b j với i > j nào đó
thì
ai p ki b i
= k = p ki −k j ¾ 2,
aj p j bj
mâu thuẫn. Vì các bi đôi một phân biệt, ta suy ra b1 b2 . . . bn ¾ n!. Suy ra

a1 a2 . . . an = p k1 +k2 +...+kn b1 b2 . . . bn ¾ p ep n! ,

trong đó e p là số mũ của p trong phân tích tiêu chuẩn của a1 . . . an . Nhân các bất đẳng
thức thu được đối với tất cả các ước nguyên tố của tích a1 a2 . . . an , ta suy ra điều phải
chứng minh.

2. Các bài toán tự giải


Bài 1 (Vũng Tàu). Cho các số nguyên dương a, b, c phân biệt. Chứng minh rằng tồn tại
số nguyên dương n sao cho a + n, b + n, c + n đôi một nguyên tố cùng nhau.
x y3
Bài 2. Bến Tre] Tìm tất cả các số nguyên dương x, y và số nguyên tố p sao cho x+ y = p.

Bài 3 (Cần Thơ). Cho a, b, c là các số nguyên. Chứng minh rằng nếu a
b + b
c + c
a là số
nguyên thì abc là lập phương của một số nguyên.
Bài 4 (Chuyên KHTN Hà Nội). Tìm tất cả n nguyên dương sao cho

n(2n + 1)2 |7n + 1.

Bài 5 (Bình Định). Giả sử phương trình x 19 + ax 2 + bx + c = 0 hệ số nguyên a, b, c có


ba nghiệm nguyên là x 1 , x 2 , x 3 . Chứng minh rằng

(a + b + c + 1)(x 1 − x 2 )(x 2 − x 3 )(x 3 − x 1 )

chia hết cho 19.

Tập san Toán học STAR EDUCATION


46

Bài 6 (Cà Mau). Cho các số nguyên a, b, x, y thỏa mãn


p p 456
a + b 123 = (x + y 123) .
Chứng minh rằng a ≥ 11b.
Bài 7 (Tiền Giang). Cho dãy số (an ) xác định bởi
2
an = (3n + (−2)n )
n
với n = 1, 2, 3, . . . Chứng minh rằng an không là số nguyên với mọi số nguyên lẻ n > 1
nhưng tồn tại vô hạn n sao cho an nguyên.
Bài 8 (Phú Yên). Cho a là số nguyên dương bất kỳ, chứng minh rằng tồn tại hai ước
nguyên tố p, q của a2021 − 1 sao cho (p − 1)(q − 1) chia hết 2021.
Bài 9 (Đồng Nai). Tìm tất cả các cặp số nguyên tố p, q sao cho p2 q2 + 3 chia hết cho
(p − 1)(q − 1).
Bài 10 (Long An). Cho dãy số (un ) xác định bởi u1 = 1 và
un
un+1 = với mọi n ≥ 1.
2 + 3 + u2n
Æ

u2n
Chứng minh rằng số u22n
− 2 có thể biểu diễn thành tổng bình phương của ba số nguyên
dương liên tiếp với mọi n ∈ Z+ .
Bài 11 (Kiên Giang). Với mỗi số nguyên dương n ≥ 2, ta viết
1 = d1 < d2 < ... < dk = n
là tất cả các ước của n theo thứ tự tăng dần. Tìm tất cả các giá trị của n sao cho 2022
là ước của n và n = d21 d22 .
Bài 12 (Quảng Trị). Với mỗi n nguyên dương, xét phương trình 3x 2 − y 2 = 23n . Chứng
minh rằng nếu n chẵn thì phương trình vô nghiệm, còn nếu n lẻ thì phương trình có
nghiệm.
Bài 13 (Ninh Thuận). Cho số nguyên tố p > 3.
1
1. Giả sử 12 + 212 + ... + (p−1)
1
2 =
m
n là phân số tối giản. Chứng minh rằng m chia hết
cho p.
p−1
2. Chứng minh rằng C5p−1 − 1 chia hết cho p3 .
Bài 14 (Gia Lai). Cho dãy số (un ) xác định bởi u1 = 3 và
nun+1 = 2(n + 1)un − n − 2 với mọi n ≥ 1.
1. Chứng minh rằng mọi số hạng của dãy đều nguyên.
2. Chứng minh rằng với mọi số nguyên tố lẻ p, tồn tại hai số hạng liên tiếp của dãy
chia hết cho p.
Bài 15 (Hậu Giang). Chứng minh rằng số 2n được viết dưới dạng 2n = a2021 + b2021
trong đó a, b, n ∈ Z+ khi và chỉ khi 2021|n − 1.
Bài 16 (Bình Thuận). Tìm nghiệm nguyên dương của phương trình
x 3 − 6x 2 + 2x y − y 3 = 0.
Bài 17 (Kon Tum). Tìm số có bốn chữ số abcd sao cho abd là số chính phương và
a bcd + 72 cũng là số chính phương.

Tập san Toán học STAR EDUCATION


Thuật toán tham lam

Hồ Quốc Nam

Thuật toán tham lam là thuật toán để giải quyết một bài toán trong đó ta chọn phương
án tối ưu cho các bước địa phương, từ đó đạt được tối ưu toàn cục. Đó chưa hẳn luôn
là phương án tối ưu cho bài toán nhưng thường sẽ được đạt được cực trị.

Ý tượng là trong mỗi bước, ta xét các phần tử (thuộc dạng tốt nhất, xấu nhất, lớn nhất
hay nhỏ nhất) để đạt được các mục tiêu cần giải quyết.

Ví dụ ta có một xấp tiền gồm các tờ 1k, 2k, 5k, 10k, làm sao ta có thể đổi một số tiền
57k thành các tờ tiền này sao cho số tờ tiền là ít nhất.
Rõ ràng ta cứ chọn các tiền lớn nhất có thể, thuật toán là
Chọn lớn nhất x ∈ {1, 2, 5, 10} thỏa x ≤ M . Tiếp tục thuật toán cho M − x.
Khi đó ta có

• x 1 = 10, M1 = 47, sau lần 1.

• x 2 = 10, M2 = 37

• x 3 = 10, M3 = 27

• x 4 = 10, M4 = 17

• x 5 = 10, M5 = 7

• x 6 = 5, M6 = 2

• x 7 = 2, M7 = 0

Vậy tập cần tìm là


10, 10, 10, 10, 10, 5, 2
Trên đây là một ví dụ đơn giản về thuật toán tham lam, tiếp theo ta xét một số ví dụ
khác để các bạn có thể hiểu rõ hơn về thuật toán này.

1. Các ví dụ

Ví dụ 4.1. Chứng minh rằng mọi số nguyên dương n thì đều có thể biểu dưới dạng
một lũy thừa của 2 hoặc một tổng các lũy thừa của 2, và sự biểu diễn là duy nhất. „

47
48

Lời giải. Nếu n là lũy thừa của 2 thì coi như xong. Nếu n không là lũy thừa của 2 ta
chọn một lũy thừa của 2 nhỏ hơn n và gần n nhất, giả sử là 2ak . Ta có 2ak < n < 2ak +1 .
Tiếp theo áp dụng thuật toán cho số n − 2k , rõ ràng thuật toán dừng vì ta có dãy giảm
và bị chặn bởi 0. Khi đó ta có n = 2a1 + · · · + 2ak .
Ta chứng minh sự biểu diễn này là duy nhất. Giả sử tồn tại hai cách biểu diễn 2ak +
· · · + 2a1 = 2 bm + · · · 2 b1 . Nếu ak > bm ⇒ ak ≥ bm + 1. Khi đó a k > a bm +1 − 1 =≥
2 bm + 2 bm −1 + · · · 21 + 1 Do đó V T > V P (vô lí)
Nếu ak = bm , chứng minh tương tự cũng có 2 dãy a1 , ·ak và b1 , · · · bm là trùng nhau.
Nên sự biểu diễn là duy nhất.

Ví dụ 4.2. Cho một đồ thị đơn, trong đó d là bậc cao nhất. Chứng minh rằng có thể
tô màu các đỉnh của đồ thị bằng d + 1 màu sao cho hai đỉnh kề thì khác màu. „

Lời giải. Giả sử các màu được đánh số từ 1 đến d + 1. Ta thực hiện cách tô màu như
sau: Chọn 1 đỉnh ta tô màu nhỏ nhất mà không xuất hiện trong các đỉnh lân cận với
đỉnh đó.
Việc tô màu này luôn thực hiện được vì bậc của mỗi đỉnh không hơn hơn d.

Ví dụ 4.3. Cho các tập A1 , A2 , · · · , A2022 là các tập con có 3 phần tử của X =
1, 2, · · · 2022. Chứng minh rằng ta có thể tô màu 674 phần tử của X sao cho với mọi
tập Ai có một phần tử không được tô màu. „

Lời giải. • Ta có thể làm ngược lại, tô màu các số sao cho với mỗi tập Ai có ít
nhất một số được tô màu, ta chứng minh là số phần tử cần tô không vượt quá
1348 số.

• Ta thực hiện cách tô sau: Chọn phần tử xuất hiện nhiều nhất trong các tập, tô
màu phần tử đó. Lặp lại thuật toán đến khi không có tập hợp nào không có
phần tử được tô màu Ta chứng minh số phần tử được tô màu không vượt quá
1348 phần tử.
Giả sử ta tô được k phần tử, và các tập còn lại đều phân biệt rời nhau, có
m tập hợp. Rõ ràng vì mỗi bước thực hiện đều giảm được ít nhất 2 tập nên
2022
k≤ = 1011. (Vì cách chọn số phần từ xuất hiện trong nhiều tập nhất nên
2
ít nhất là 2). Và các tập còn lại phân biệt rời nhau, mỗi tập có 3 phần tử nên
m ≤ 1011/3 = 337.
Khi đó ta tô màu mỗi phần tử trong m tập còn lại thì sẽ thỏa đề bài, do đó số
phần tử cần tô màu là k + m ≤ 1011 + 337 = 1348.
Do đó ta có cách tô màu thỏa đề bài.

Ví dụ 4.4. Cho một bảng 2 × n, người ta điền vào bảng các số dương sao cho tổng hai
số trong cùng một cột bằng 1. Chứng minh rằng ta có thể chọn mỗi cột một số sao cho
n+1
các số trên mỗi dòng không lớn hơn . „
4

Tập san Toán học STAR EDUCATION


49

Lời giải. Ta sắp xếp từ trái sang phải theo thứ tự tăng dần các số ở hàng thứ nhất
a1 ≤ a2 ≤ · ≤ an , hàng dưới tương ứng là 1 − a1 , 1 − a2 , · · · , 1 − an .
Ta chọn các số ở hàng trên từ trái qua, nhiều nhất sao cho tổng S không lớn hơn
n+1
. Ta chứng minh tổng các số ở hàng dưới trong các cột còn lại không lớn hơn
4
n+1
− S.
4
Giả sử chỉ số i lớn nhất thỏa a1 + a2 + · · · + ai ≤ n+1
4 . Ta cần chứng minh

n+1
≥ (1 − ai+1 ) + (1 − ai+2 ) + · · · + (1 − an ) = (n − i) − (ai+1 + ai+2 + · · · + an )
4

3n − 1
ai+1 + ai+2 + · · · + an ≥ −i
4
Từ a1 ≤ a2 ≤ · · · ≤ ai+1 , ta có
a1 + a2 + · · · + ai+1
≤ ai+1
i+1
và từ ai+1 ≤ ai+2 ≤ · · · ≤ an , ta có
ai+1 + ai+2 + · · · + an
≥ ai+1
n−i
Từ đó ta có
ai+1 + ai+2 + · · · + an a1 + a2 + · · · + ai+1

n−i i+1
n+1
a1 + a2 + · · · + ai+1 4
ai+1 + ai+2 + · · · + an ≥ (n − i) · > (n − i) ·
i+1 i+1
do cách chọn i lớn nhất.
n − i n + 1 3n − 1
Ta cần chứng minh ≥ − i.
i+1 4 4 p
hay (n − 1)2 ≥ 4i(n − i − 1), áp dụng AM-GM 2 i(n − i − 1) ≤ i + (n − i − 1) = n − 1,
ta có điều cần chứng minh.

Ví dụ 4.5. (IMOSL 2001) Một tập có 3 số nguyên phân biệt không âm {x, y, z} thỏa
x < y < z được gọi là tập có tính chất P nên {z − y, y − z} = {a, b} với 0 < a < b
nguyên dương cho trước. Chứng minh rằng tập các số nguyên không âm được viết
thành hợp các có tính chất P đôi một phân biệt. „

Lời giải. Ta thấy có 2 trường hợp là z = y = b, y − x = a hoặc z − y = a, y − x = b


hay tập có tính chất P sẽ có một trong hai dạng x, x+a, x+a+b hoặc x, x+b, x+a+b
với x nguyên dương.
Với mỗi tập có tính chất P là x, y, z ta tô màu x đỏ, y xanh z vàng.
Ta tô màu các số nguyên dương như sau:
Xét số dương k nhỏ nhất chưa được tô màu. Nếu k + a chưa được tô màu thì ta tô
màu cho tập k, k+a, k+a+b. Nếu k + b chưa được tô màu thì ta tô màu cho tập k,
k+b,k+a+b.

Tập san Toán học STAR EDUCATION


50

Ta chứng minh với cách tô màu như trên thì các tập đều phân biệt và số nào cũng
được tô màu. Giả sử ta đã thực hiện tới bước thứ n với phần tử x n là nhỏ nhất được
tô màu. Ta tiếp tục thực hiện với kế tiếp với phần tử x n+1 chưa được tô màu. Ta cần
chứng minh các phần tử x n+1 + a hoặc x n+1 + b chưa được tô màu và x n+1 + a + b chưa
được tô màu trước đó.
Thật vậy rõ ràng là x n+1 + a + b chưa được tô màu vì nó lớn hơn tất cả các số được tô
màu. (Ở bước thứ n thì x n được tô màu, nên x n + a + b là số lớn nhất được tô màu.
Nếu x n+1 + b được tô màu thì không thể là màu đỏ, nếu xanh thì x n+1 + b − a được tô
màu đỏ, vô lí. Nếu màu vàng thì x n+1 + b − a − b = x n+1 − a được tô màu đỏ, khi đó
x n+1 được tô màu, vô lí.

2. Bài tập
Bài 1. (Đức 2000) Có một số đá tổng cộng là 9 tấn cần được vận chuyển bằng xe tải.
Mỗi tảng đá không nặng quá 1 tấn, mỗi xe tải chở không quá 3 tấn. Hỏi còn ít nhất bao
nhiêu xe tải để chở hết số đá này cùng lúc.

Bài 2. Chứng minh rằng với mọi số nguyên dương n tồn tại các số nguyên dương
a1 , a2 , · · · , sao cho ai ≤ i thỏa

n = a1 · 1! + a2 · 2! + · · · + ak · k!
và sự biểu diễn này là duy nhất.

Bài 3. Gọi S là tập hợp n điểm trong mặt phẳng tọa độ. Nói rằng một cặp điểm thẳng
hàng nếu hai điểm đó có cùng tọa độ x hoặc y. Chứng minh rằng S có thể được phân
chia thành các tập con rời rạc sao cho

• Mỗi tập con này là một tập hợp các điểm thẳng hàng,

• Tối đa n3/2 cặp điểm phân biệt không có thứ tự trong S thẳng hàng nhưng không
nằm trong cùng một tập con.

Bài 4. Cho n là một số nguyên dương. Tìm số nguyên k nhỏ nhất thỏa mãn tính chất:
Cho bất kỳ số thực a1 , , ad sao cho a1 + a2 + + ad = n và 0 ≤ ai ≤ 1 với i = 1, 2, , d, có
thể phân chia các số này thành k nhóm (một số có thể trống) sao cho tổng các số trong
mỗi nhóm nhiều nhất là 1.

Bài 5. (IMO 2014). Với mỗi số nguyên dương n, Ngân hàng Cape Town phát hành tiền
1
xu có mệnh giá . Đưa ra một bộ sưu tập hữu hạn các đồng tiền như vậy (không nhất
n
1
thiết phải có mệnh giá khác nhau) với tổng số giá trị nhiều nhất là 99 + . Chứng minh
2
rằng có thể chia bộ sưu tập này thành 100 nhóm hoặc ít hơn, chẳng hạn như mà mỗi
nhóm có tổng giá trị nhiều nhất là 1.

Tập san Toán học STAR EDUCATION


Đề thi chọn đội tuyển toán
Trường PTNK năm học 2021

(Nguyễn Ngọc Duy - Nguyễn Tăng Vũ)

Ngày thi thứ nhất


Thời gian làm bài: 180 phút
——————

Bài 1. Tìm hàm số f : R → R thỏa f (x f ( y) + f (x)) = f (x) + x y + x + 1, ∀x, y ∈ R.


s
un un−1
Bài 2. Cho dãy số (un ) thỏa u1 = 2, u2 = 1 và un+1 = với mọi n ≥ 2. Xét dãy
n
số (vn ) xác định bởi vn := u1 + u2 + . . . + un , ∀n ≥ 1. Chứng minh dãy (vn ) hội tụ.

Bài 3. Cho p là số nguyên tố, n là số nguyên dương thỏa 2 < p < n. Gọi A là tập hợp
các đa thức P(x) = x n + an−1 x n−1 + . . . + a1 x + a0 có tất cả các hệ số thuộc tập
{1; 2; . . . ; n!} và P(m) chia hết cho p với mọi số nguyên dương m.

(a) Chứng minh tổng a1 + a p + a2p−1 + . . . + a1+k(p−1) chia hết cho p với k =
n−1
• ˜
(xem an = 1 ), kí hiệu [x] là phần nguyên của x.
p−1
(b) Tính số phần tử của A theo n và p.

Bài 4. Cho tam giác ABC có (I) là đường tròn nội tiếp. Một đường thẳng qua A cắt (I)
tại M , N . Gọi T là giao điểm của các tiếp tuyến với (I) tại M , N .

(a) Chứng minh rằng nếu AT k BC thì M N đi qua trung điểm của BC.
(b) Gọi D là tiếp điểm của (I) với AB và E là giao điểm của DM với AC. Trên
EN lấy điểm F thoả T F vuông góc AI. Chứng minh rằng khi đường thẳng
AM N thay đổi, giao điểm P của M F và DN thuộc một đường thẳng cố
định.

51
52

Ngày thi thứ hai


Thời gian làm bài: 180 phút
——————

Bài 5. Cho n số thực x 1 , x 2 , . . . , x n thỏa hiệu giữa số lớn nhất và số nhỏ nhất của chúng
là 1 . Ta xây dựng

x1 + x2 x1 + x2 + . . . + x n
y1 = x 1 , y2 = , . . . , yn =
2 n

Gọi M , m lần lượt là số lớn nhất và nhỏ nhất trong các số y1 , y2 , · · · , yn .


Tìm giá trị lớn nhất của M − n.

Bài 6. Cho tập X = {1; 2; . . . ; 20}. Tập con A của X được gọi là tập ""tránh 2" " nếu với
mọi x, y thuộc A thì |x − y| khác 2 . Tìm số các tập con ""tránh 2" " của X có 5
phần tử.

Bài 7. Cho tam giác ABC và điểm D trên cạnh BC. Các đường tròn ( ABD ), ( AC D )
lần lượt cắt AC, AB tại E, F . Gọi I là tâm đường tròn (AE F ). a) Chứng minh I D
vuông góc BC. b) Gọi H là giao điểm của I D với E F và K là điểm thoả mãn
H BK = H C K = 90◦ . Các đường tròn (I BK), (I C K) lần lượt cắt I C, I B tại M , N .
Chứng minh tâm J của đường tròn (I M N ) thuộc trung trực BC.

Bài 8. Cho p là số nguyên tố. Với mọi số nguyên a, đặt

q := 1 + a + a2 + . . . + a p−1 .
 
Chứng minh (1 − a) 1 − a2 . . . 1 − a p−1 − p chia hết cho q.

Lời giải

Bài 1. Cho x = 1 ⇒ f ( f ( y) + f (1)) = f (1) + y + 2, suy ra f là song án, khi đó tồn tại
a ∈ R sao cho f (a) = 0.
Trong (1) cho x = y = a ⇒ f (0) = a2 + a + 1, (2)
Trong (1) cho§x = a ⇒ f (a f ( y)) = a y + a + 1, (3).
f (0) = 0
Nếu a = 0 ⇒ (vô lí ).
f (0) = 1
Nếu a 6= 0 trong (3) chọn y = −a−1 −a−1

a ⇒ f a f a  = 0 = f (a)
−a−1 −a−1
Do f là song ánh suy ra a f a =a⇒ f a = 1.
−a−1

Trong (3) cho y = −1 ⇒ f (a f (−1)) = 1 ⇒ f a = f (a f (−1)) = 1.
Cũng do f là song ánh suy ra a = a f (−1) ⇒ f (−1) = −a−1
−a−1
a2 , (4).
Trong (1) cho x = y = 1 ⇒ f (0) = f (−1) + 1, (5).
Từ (2), (4) và (5) ta suy ra a2 + a + 1 = −a−1

a2 + 1 ⇔ (a + 1) a2 − a + 1 = 0 ⇔
2

a = −1 ⇒ f (−1) = 0.
Trong (1) cho y = −1 ⇒ f ( f (x)) = f (x) + 1, (6)
Do f là song ánh nên ∀t ∈ R tồn tại x 0 ∈ R sao cho f (x 0 ) = t.
Trong (6) cho x = x 0 ⇒ f (t) = t + 1, ∀t ∈ R.
Vậy f (x) = x + 1, ∀x ∈ R.

Tập san Toán học STAR EDUCATION


53

Bài 2. Dễ thấy (un ) là dãy số giảm nên với mọi số tự nhiên n > 7 thì
s
un−1 un−2 un−2 un−4 2 2
un = <p <p < ... < p < .
n−1 n−1 (n − 1)(n − 3) (n − 1)(n − 3) . . . (n − 3)(n − 7)
Nên: € Š
2
vn = u1 +u2 +. . .+un < u1 +u2 +. . .+u7 + (n−3)(n−7) + . . . < u1 +u2 +. . .+u7 +1.
Suy ra (vn ) là dãy tăng và bị chặn nên nó hội tụ.

Bài 3. (a) f (m) ≡ a0 + (a1 + a p + a2p−1 + · · · )m+


 
a2 + a p+1 + a2p + · · · m2 + . . . a p−1 + a2p−2 + a3p−3 + · · · m p−1 ≡ 0(modp)
Áp dụng định lý Lagrange suy ra câu a) .
(b) Từ câu a) ta có
 
• a0 ; a1 + a p + a2p−1 + · · · . ; . . . . . . ; a p−1 + a2p−1 + a3p−3 + · · · là các bội
của p.
(n!)k
• Với mỗi q thuộc {0; 1; 2; . . . ; p − 1} có p bộ (x 1 ; x 2 ; . . . ; x k ) thuộc
{1; 2; . . . ; n!}k sao cho x 1 + x 2 + · · · + x k ≡ q(modp).
k
(n!)k1 (n!) p−1 (n!)n
• Xét q = 0:|A| = n!
p · p ...... p = pp với k1 , k2 . . . , k p−1 lần lượt
là số phần tử của các tập
  
a1 ; a p ; a2p−1 ; . . . , a2 ; a p+1 ; a2p ; . . . .. . . . . , a p−1 ; a2p−2 ; a3p−3 ;

Bài 4. (a) Gọi tiếp điểm của (I) với AB, AC lần lượt là D, K. Khi đó tứ giác DM K N là
tứ giác điều hòa, nên T thuộc DK.
Mặt khác A(M T ; BC) = −1, mà AT k BC nên AM đi qua trung điểm BC.
A

E
M T
K
F
Q
D

I
P
N

B S C

(b) Gọi P là giao điểm DN và M F ; Q là giao điểm của DT và AM .Tứ giác


DM K N là tứ giác điều hòa, DT vuông góc với AI nên F thuộc T D. Ta có
(AQ; M N ) = −1, suy ra E(AQ; M N ) = −1, suy ra E(KQ; DF ) = −1. (1)
Mặt khác xét tứ giác DM F N thì E(PQ; DF ) = −1 (2)
Từ (1) và (2) suy ra E, K, P thẳng hàng, từ đó P thuộc đường thẳng AC cố
định.

Tập san Toán học STAR EDUCATION


54

Bài 5. Nhận xét : Hiệu của hai số bất kì trong dãy y1 , . . . , yn sẽ không thay đổi nếu ta
tịnh tiến các số x 1 , x 2 , . . . , x n thành x 1 − c, x 2 − c, . . . , x n − c ( c là một số nào
đó). Do đó nếu chọn c = min x k thì dãy x 1 , x 2 , . . . , x n sẽ tịnh tiến thành dãy các
số mới với min x k = 0 và max x k = 1. Ta sẽ xét bài toán với dãy x 1 , . . . , x n có
min x k = 0 và max x k = 1. Giả sử m = mink yk = y p và M = maxk yk = yq . Ta
xét hai trường hợp:
Trường hợp 1: p < q
x1 + . . . + xq x1 + . . . + x p
yq − y p = −
q p
  1 1 ‹ x p+1 + . . . + x q q−p p 1
= x1 + . . . + x p − + ≤ =1− ≤1−
q p q q q n
1 1
(Vì x 1 + . . . + x p ≥ 0, − < 0 và x p+1 + . . . + x q ≤ q − p )
q p

 x1 = x2 = . . . = x p = 0
Dấu “ = "xảy ra ⇔ x p+1 = . . . = x q = 1 , nghĩa là dãy các số x 1 , . . . , x n là 0, 1, 1, . . . , 1
 p = 1, q = n

Trường hợp 2: p > q


x1 + . . . + xq x1 + . . . + x p
yq − y p = −
q p
  1 1 ‹ x q+1 + . . . + x p 
1 1
‹
q 1
= x1 + . . . + xq − − ≤q − =1− ≤1−
q p p q p p n
x +...+x p
(Vì x q+1 + . . . + x p ≥ 0 ⇒ − q+1 p ≤ 0 và x 1 + . . . + x q ≤ q ) Dấu " = "xảy ra ⇔

 x q+1 = x q+2 = . . . = x p = 0
x1 = . . . = xq = 1 , nghĩa là dãy các số x 1 , . . . , x n là 1, 0, 0, . . . , 0
 q = 1, p = n
Vậy Tmax = 1 − 1n

Bài 6. Ta gọi tập X là tập ""tránh 1"" nếu hiệu hai phần bất kì của X là khác 1.

Nếu A là một tập con "tránh 2" của X thì xét A1 = {x | 2x ∈ A}, A2 = {x |
2x − 1 ∈ A} thì rõ ràng A1 , A2 là các tập con "tránh 1" của Y = {1, 2, 3, . . . , 10}
và |A1 | + |A2 | = |A|. Ngược lại, nếu A1 , A2 là các tập con "tránh 1" của Y thì
(2A1 ) ∪ (2A2 − 1) là một tập con "tránh 2" của X . Với mỗi k = 1, . . . , 5, ta chứng
k
minh số tập con "tránh 1" có k phần tử của Y bằng C11−k . Thật vậy, giả sử
{ y1 , . . . , yk } là một tập con "tránh 1" của Y thì đặt

x 1 = y1 − 1, x 2 = y2 − y1 − 1, . . . , x k = yk − yk−1 − 1, x k+1 = 10 − yk .

Ta sẽ có
x 1 + x 2 + . . . + x k+1 = 10 − k
x 1 , x k+1 ≥ 0, x 2 , . . . , x k ≥ 1
Ngược lại, nếu ta có 1 bộ nghiệm của (∗) thì ta dựng lại được một tập con k
phần tử "tránh 1" của Y bằng cách đặt y1 = x 1 + 1, y2 = x 2 + x 1 + 2, . . . , yk =

Tập san Toán học STAR EDUCATION


55

x k +x k−1 +. . .+x 1 +k. Để tính số nghiệm của (∗), t a đặt z2 = x 2 −1, . . . , zk = x k −1


và đưa về phương trình
x 1 + z2 + . . . + zk + x k+1 = 11 − 2k với
x 1 , z2 , . . . , zk , x k+1 nguyên không âm. Theo kết quả bài toán chia kẹo Euler, số
k
nghiệm nguyên không âm của phương trình này bằng C11−k . Sử dụng kết quả
này và lập luận ở đầu lời giải, ta suy ra số tập con "tránh 2" có 5 phần tử của X
bằng 
2 C65 + C74 C10
1
+ C83 C92 = 4744.

Bài 7. (a) Ta có ∠C DE = ∠BAC, ∠DBF = ∠BAC, suy ra ∠C DE + ∠BDF = 2∠BAC =


∠E I F , suy ra ∠E I F + ∠E DF = 180◦ , tứ giác I E DF nội tiếp.
Từ I E = I F và ∠BDF = ∠C DE nên DI, DB là phân giác trong và phân giác
ngoài của ∠E DF , suy ra I D⊥BC.

H E

B C
D
N

(b) Ta có tam giác DC E và DBF đồng dạng, suy ra DE · DF = DB · DC.


Mặt khác tam giác DH E và DF I đồng dạng, suy ra DH · DI = DE · DF .
Từ đó DH · DI = DB · DC, mà AD⊥BC, suy ra H là trực tâm tam giác I BC,
từ đó ta có I BKC là hình bình hành.
Khi đó các tứ giác I C K M và I BK N là hình thang cân, trung trực của I M
cũng là trung trực C K nên qua trung điểm H K, tương tự trung trực của
I N cũng qua trung điểm H K, do đó tam ngoại tiếp tam giác I M N là trung
điểm H K cũng là tâm ngoại tiếp tứ giác H BKC, do đó thuộc trung trực
BC.

Bài 8. Xét số phức đơn vị e p = e2πi/p . Suy ra đa thức hệ số nguyên


€ Š € Š
P(x) = 1 + x + x + . . . + x
2 p−1
= x − ep x − ep . . . x − ep
2 p−1

Tập san Toán học STAR EDUCATION


56

 
Xét đa thức hệ số nguyên: Q(x) = (1 − x) 1 − x 2 . . . 1 − x p−1 Ta nhận xét,
với mọi số nguyên 1 ≤ j < p thì
€ Š € Š€ Š € Š
Q e pj = 1 − e pj 1 − e2p j . . . 1 − e(p−1)
p
j

  
= 1−e i2π· j/p
1−e i2π·2 j/p
. . . 1 − e i2π·(p−1) j/p
  
= 1 − e i2π/p 1 − e i2π2/p . . . 1 − e i2π(p−1)/p
  
= x − e p x − e p 2 . . . x − e p p−1 = P(1) = p

(Điều này có được do { j, 2 j, . . . ., (p − 1) j} là hệ thặng dư đầy đủ theo modul p )


Do đó đa thức Q(x) − p sẽ có các nghiệm là e p j , 1 ≤ j ≤ p − 1. Nên : Q(x) − p =
P(x)H(x). Bằng đồng nhất hệ số ta thấy H(x) cũng là đa thức nguyên. Thay
x = a vào ta suy ra
.
(1 − a) 1 − a2 . . . 1 − a p−1 − p = P(a)H(a)..q
 

Tập san Toán học STAR EDUCATION


Đề thi chọn đội dự tuyển
Trường PTNK năm 2022

Thời gian làm bài: 120 phút

Bài 1. Cho a, b, c ≥ 0 thỏa a2 + b2 + c 2 = 1. Tìm giá trị lớn nhất và giả trị nhỏ nhất của
biểu thức P = ab + bc + ca − 2(a + b + c).

Bài 2. Cho k, n ∈ Z + thỏa k < n . Tính số đơn ánh f : {1, 2, 3, . . . , 2k+1} → {1, 2, 3, . . . , 2n}
thỏa mãn điều kiện f (1) < f (2) < . . . < f (k) < f (k + 1) > f (k + 2) > . . . >
f (2k) > f (2k + 1) và f (k + 1) 6= 2n − 2.

Bài 3. Cho n là số nguyên dương, kí hiệu a(n) = 1+2+. . .+n và b(n) = 12 +22 +. . .+n2 .
Hỏi có tồn tại số n sao cho 2(n + 1)a(n) + 3b(n) − 1 là số chính phương?

Bài 4. Cho tam giác ABC có 2A = 5B = 10C. Phân giác trong BD cẳt trung tuyển C M
tại I. Một đường thẳng d đi qua D vuông góc với AC cắt BC và AI lần lượt tại E
và K. AE cắt C K tại F . Chứng minh: M F song song BK.

Lời giải

Bài 1. Đặt t = a + b + c ta có a(1 − a) ≥ 0, b(1 − b) ≥ 0, c(1 − c) ≥, suy ra a + pb + c ≥


a2 + b2 + c 2 = 1, và (a + b + c)2 ≤ 3(a2 + b2 + c 2 ) = 3, suy ra a + b + c ≤ 3 Ta có
t2 − 1
1 = (a + b + c)2 − 2(ab + bc + ac) ⇒ ab + bc + ca = .
2
t2 − 1 1 1 p
Do đó P = − 2t = t 2 − 2t − với 1 ≤ t ≤ 3.
2 2 2 p
Khảo sát hàm bậc hai trong đoạn ta có max P = −2 khi t = 1 và min P = 1 − 2 3.
p 1
Vậy max P = −2 khi a = 1, b = c = 0 và min P = 1 − 2 3 khi a = b = c = p .
3
Bài 2. Do đó f là đơn ánh, I m f là một tập con có 2k + 1 phần tử của A, mặt khác
f (k + 1) là giá trị lớn nhất nên I m f có giá trị lớn nhất khác 2n − 2.
Ta đếm số tập con có 2k + 1 phần tử của A mà phần tử lớn nhất khác 2n − 2.
Số tập con có 2k + 1 của A là C2n 2k+1
, số tập con có 2k + 1 mà có phần tử lớn nhất 2n − 2
2k
là bằng với số tập con có 2k phần tử của {1, 2, · · · 2k − 3}, là C2n−3 .
Do đó theo nguyên lí bù trừ số tập con có 2k + 1 của tập A mà phần tử lớn nhất khác
2n − 2 là (C2n2k+1 2k
− C2n−3 ).
Tiếp theo ta đếm số đơn ánh từ {1, 2, · · · , 2k + 1} tới A0 = {a1 , a2 , · · · , a2k+1 } thỏa đề
bài, ta có f (k + 1) = a2k+1 , nên số đơn ánh bằng số cách chọn k phần tử từ A0 nên
k
bằng C2k .
Vậy số đơn ánh thỏa đề bài C2k k
(C2n
2k+1 2k
− C2n−3 )

57
58

n(n + 1) n(n + 1)(2n + 1)


Bài 3. Ta có a(n) = , b(n) =
2 6
n(n + 1)(4n + 3)
Khi đó P(n) = 2(n + 1)a(n) + 3b(n) − 1 = − 1.
2
Giả sử P(n) là số chính phương ta có n(n+1)(4n+3) = 2(x 2 +1), ta có n(n+1)(4n+3)
luôn có ước nguyên tố dạng p = 4k + 3, suy ra p|2(x 2 + 1) suy ra p|x, p|1, vô lí!
Vậy không tồn tại n để P(n) là số chính phương.
Bài 4.

D
M
I

45◦
E
B C
N

5π π π
Ta tính được ∠A = , ∠B = , ∠C = . Vẽ đường cao AN , N thuộc BC.
8 4 8
BN AN sin C AD AB sin C BN AD
Ta có = = và = = , sin 5C = cos C,suy ra = ,
NC NC cos C CD BC sin 5C NC CD
do đó AN , BD, C M đồng quy tại I và DN k AB.
Ta có ∠BAN = ∠AN D = ∠AC K = 2∠AC K, suy ra AC K cân và N là trung điểm AK, từ
đó tam giác ABK vuông cân.
Khi đó ∠F N K = ∠AC K = 45◦ = ∠AK B và ∠AN M = 45◦ , do đó M , N , F thẳng hàng và
M F k BK.
Lời giải được thực hiện bởi nhóm trợ giảng chuyên toán của Star Education.

Tập san Toán học STAR EDUCATION


Đề kiểm tra lớp 10 Chuyên đề

Phạm Nguyễn Mạnh

Thời gian làm bài: 180 phút

Bài 1. Với n > 1 nguyên dương, cho các số thực

0 ≤ x 1 ≤ x 2 ≤ · · · ≤ x n và 0 ≤ y1 ≤ y2 ≤ · · · ≤ yn

thỏa mãn x 1 + x 2 + · · · + x n = y1 + y2 + · · · + yn = 1. Tìm giá trị lớn nhất của

(a) min {|x 1 − y1 | , |x 2 − y2 | , . . . , |x n − yn |}.


(b) P := |x 1 − y1 | + |x 2 − y2 | + · · · + |x n − yn |.

Bài 2. Tìm tất cả các hàm số f : R → R thỏa mãn:

1 1
 ‹ ‹
f (x + x y + f ( y)) = f (x) + f ( y) + , ∀x, y ∈ R.
2 2

Bài 3. Từ các số 1, 2, 3, 4, 5 có thể lập được bao nhiêu số tự nhiên có 2022 chữ số mà

(a) Không có số 1 hoặc không có số 2.


(b) Trong mỗi số đó chứa một số lẻ chữ số 1 và một số chẵn chữ số 2?

Bài 4. Cho tam giác ABC nhọn có đường cao AH. Đường tròn nội tiếp (I) tiếp xúc với
BC, CA, AB lần lượt tại D, E, F . Đường tròn (A) có tâm A bán kính AE cắt đọn
thẳng AH tại điểm K. Đưòng thẳng I K cắt đưòng thẳng BC tại P. Các đuòng
thẳng DK và P K cắt đường tròn (A) lần lượt tại Q và T khác K.

(a) Chứng minh rằng tứ giác T DPQ nội tiếp và ba điểm Q, A, P thẳng hàng.
(b) Đường thẳng DK cắt (I) tai điểm thứ hai là X . Chứng minh rằng ba đường
thẳng AX , E F, T I đồng quy.

Bài 5. Tìm tất cả các giá trị hữu tỉ a sao cho tồn tại 2 đa thức P(x), Q(x) có hệ số
nguyên và hệ số bậc cao nhất là 1 thỏa mãn:
 
P x2 P(x) 2
‹
= −a
Q (x 2 ) Q(x)

Hết

59
60

Lời giải

Bài 1.
1
1. a) Ta có x 1 + x 2 + · · · + x n = 1, x 1 = min {x 1 , x 2 , . . . , x n }, nên x 1 ≤
. Tương tự
n
1 1
y1 ≤ . Do đó ta luôn có |x 1 − y1 | ≤ , hay min {|x 1 − y1 | , |x 2 − y2 | , . . . , |x n − yn |} ≤
n n
1
, đẳng thức có thể xảy ra khi ta lấy
n
x1 = x2 = · · · = x n, y1 = y2 = · · · = yn−1 = 0, yn = 1

1
tức là giá trị lớn nhất cần tìm là .
n
b) Ta có

|x − y| = max{x, y} − min{x, y} = x + y − 2 min{x, y}

Sử dụng hằng đẳng thức này, ta có


n
X n
X
S= |x i − yi | = (min {x i , yi )
i=1 i=1
n
X
=2−2 min {x i , yi } ≤ 2 − 2 min {x n , yn }
i=1

Chú ý rằng do x n = max {x 1 , x 2 , . . . , x n }, và yn = max { y1 , y2 , . . . , yn } nên x n , yn ≥


1
.
n
2
Do đó S ≤ 2 − . Đẳng thức có thể đạt được với các dãy (x i ) , ( yi ) đã trình bày
n
như trong câu a).

Bài 2. Dễ thấy hàm f hằng không ‹  thỏa mãn. ‹ Ta xét f không hằng.
1 1
f (x + x y + f ( y)) = f (x) + f ( y) + , ∀x, y ∈ R (1)
2 2
1 1
 ‹ ‹
Trong (1) cho y = −1 ta được: f ( f (−1)) = f (x) + f (−1) + , ∀x ∈ R(2)
2 2
1 1 1
Rõ ràng nếu f (−1) + 6= 0 thì f là hàm hằng. Do đó: f (−1) + = 0 ⇒ f (−1) = −
2 2 2
1
Ta sẽ chứng minh: f (x) + = 0 ⇔ x = −1.
2
−1
Thật vậy, giả sử tồn tại a 6= −1 sao cho f (a) = .
‹ 2
1

Trong (1) chọn y = a ta có: f ax + x − = 0, ∀x ∈ R. Mâu thuẫn vì f không là
2
hàm hằng. Do đó ta có: a = −1.
1
− − f ( y)
1 −1
 ‹
Chú ý là f (−1) = − nên từ (2) ta có : f = 0. Trong (1) chọn x = 2 , ( y 6=
2 2 1+ y

Tập san Toán học STAR EDUCATION


61

−1) ta được:

1 1 1
     
− − f ( y) − − f ( y) − − f ( y)
 1 1
 ‹
 2 2    2
f + · y + f ( y) = f   +  f ( y) +
1+ y 1+ y 1+ y 2 2

1
   
− − f ( y)
 1 1 1
 ‹  ‹
  2
⇔ f   +  f ( y) + =f − = 0, ∀ y 6= −1
1+ y 2 2 2

1 1
 
− − f ( y) − − f ( y)
1
⇒f 2  = − , ∀ y 6= −1 ⇔
2 = −1, ∀ y 6= −1
 
1+ y 2 1+ y

1 1
Do f (−1) = − nên f (x) = x + , ∀x ∈ R. Thử lại ta có hàm số cân tìm là f (x) =
2 2
1
x + , ∀x ∈ R.
2
Bài 3.

a) Số các số là 42022 + 42022 − 32022 = 2 · 42022 − 32022 .

b) Tổng số các chữ số của 1 và 2 là một số lẻ n ≤ 2022.

n
• Số cách chọn vị trí của các chữ số 1, 2 là C2022 .

• Số cách chọn các số 3, 4, 5 là 32022−n .

• Số cách chọn số 1, 2 vào n vị trí là Cn1 + Cn3 + · · · + Cnn = 2n−1 .

• Do đó số các số là

X 1  52022 − 1
n
C2022 · 2n−1 · 32022−n = (3 + 2)2022 − (3 − 2)2022 =
n mod 2=1
2 4

Bài 4.

Tập san Toán học STAR EDUCATION


62

X
T
E
F K
I

B H D P C

1
a) Ta có ∠H P T = 90◦ − ∠H K P và ∠TQK = ∠TAK = 90◦ − ∠AK T = ∠H P T , suy
2
ra DTQP nội tiếp.

Ta có I K · I T = I E 2 = I D2 , suy ra 4I K D ö 4I DT , do đó ∠I DT = ∠I K D =
∠QK T = 90◦ − ∠AQT .

Suy ra ∠AQT = 90◦ − ∠I DT = ∠T DH = ∠PQT . Do đó A, Q, P thẳng hàng.

b) I K · I T = I E 2 = I X 2 , suy ra ∠I T X = ∠I X D = ∠I DX = ∠I K X . Do đó AK là tiếp
tuyến của (T X K), hơn nữa AT = AK do đó AT cũng là tiếp tuyến của (T X K).

Xét 2 đường tròn (T X D) và (I) thì AX là trục đẳng phương vì AT = AE.

Khi đó xét các đường tròn (I), (T X K), (A) thì các trục đẳng phương AX , T K, F E
đồng quy.

Bài 5. Dễ thấy ta có thể giả sử 2 đa thức nguyên tố cùng nhau (nếu không thì chia
P(x), Q(x) cho UCLN). Ta có:
  
P x 2 Q2 (x) = Q x 2 P 2 (x) − aQ x 2
 
⇒ Q x 2 | P x 2 Q2 (x)

⇒ Q x 2 | Q2 (x)

Tập san Toán học STAR EDUCATION


63

 
Mà Q x 2 , Q2 (x) có bậc bằng nhau nên Q x 2 = Q2 (x) Từ đây ta chứng minh được
Q(x) = x n với n là số tự nhiên.

⇒ P x 2 = P 2 (x) − ax 2n
Cho P(x) = x m + am−1 x m−1 + . . . + a1 x + a0

Đồng nhất hệ số 2 vế của PT trên ta sẽ suy ra được a = 0 hoặc a = 2 và n = 2 m

Tập san Toán học STAR EDUCATION


Một số bài tập rèn luyện

1. Bài tập đại số-giải tích


Bài 1. (1)Cho các số không âm a, b, c thỏa ab + bc + ca = 1. Chứng minh rằng

1 1 1
 ‹
4≤ p +p +p (a + b + c − abc).
1 + a2 1 + b2 1 + c2

Bài 2. (1) Cho các số thực a1 , a2 , . . . , an thỏa a1 + a2 + . . . + an = 0. Chứng minh rằng


với a ≥ 0 thì
a + a12 + a22 + . . . + an2 ≥ m (|a1 | + |a2 | + . . . + |an |)
Æ Æ
với m = 2 na , khi n chẵn, và m = 2 n2an−1 , khi n lẻ.

Bài 3. (1)Cho n là số nguyên dương, chứng minh


n−1
X 1 2 2 
= n −1
2 kπ 3
k=1 cos 2n

Bài 4. (1) Cho P(t) là một đa thức hệ số nguyên sao cho P(1) = P(−1). Chứng minh
rằng tồn tại đa thức Q(x, y) hệ số nguyên sao cho P(t) = Q(t 2 − 1, t(t 2 − 1))

Bài 5. (1) Một đa thức hệ số nguyên được gọi là có tính chất T nếu có thể biểu diễn
thành tổng lập phương một hay nhiều đa thức hệ số nguyên. Ví dụ 9x 3 − 3x 2 + 3x + 7 =
(x − 1)3 + (2x)3 + 23 có tính chất T . Trong các đa thức sau, đa thức nào có tính chất T ?

1. P(x) = 3x 7 + 3x;

2. Q(x) = 3x 2008 + 3x 7 + 3x.

Bài 6. Tìm tất cả các hàm f : R → R sao cho với mọi đường thẳng d thì số giao điểm
của d và đồ thị hàm số f bằng số giao điểm của d và parabol y = x 2 .

Bài 7. Cho một họ hữu hạn các đa thức có đồ thị lần lượt là H1 , H2 , ..., H n . Chứng minh
rằng tồn tại một đa thức có đồ thị không cắt H i với mọi i = 1, 2, ..., n.

Bài 8. Cho hai đa thức P(x) = ax 2 + bx + c và Q(x) = Ax 2 + B x + C. Giả sử |P(x)| ≤


|Q(x)|, ∀x ∈ R, chứng minh rằng |b2 − 4ac| ≤ |B 2 − 4AC|.

Bài 9. Cho A là một tập con hữu hạn của R, giả sử f : A → A là một ánh xạ thỏa điều
kiện | f (x) − f ( y)| < |x − y|, ∀x, y ∈ A, x 6= y. Chứng minh rằng tồn tại a ∈ A sao cho
f (a) = a.

64
65

Bài 10. Cho p là số nguyên tố. f là một đa thức với hệ số nguyên sao cho f (0) =
0, f (1) = 1 và với mọi số tự nhiên n thì f (n) ≡ 0( mod p) hoặc f (n) ≡ 1( mod p).
Chứng minh rằng d ≥ p − 1. Tìm một đa thức thỏa đề bài.

Bài 11. Có tồn tại hay không một dãy vô hạn các số a1 , a2 , ..., an , ... sao cho với mọi n
thì đa thức P(x) = a0 + a1 x + a2 x 2 + ... + an x n có đúng n nghiệm thực phân biệt.

Bài 12. Cho đa thức P(x) = 2x 3 − 9x 2 + 16x − 9. Gọi x i là nghiệm của phương trình
P(x) = i với i = 0, 1, 2, 3. Tính x 0 + x 1 + x 2 + x 3 .

Bài 13. Cho đa thức P(x) = an x n + an−1 x n−1 + ... + a1 x + a0 , trong đó an nguyên dương
và các hệ số còn lại là các số nguyên. Đặt S(n) là tổng tất cả các chữ số của n. Chứng
minh rằng dãy S(P(n)) có số lặp lại vô hạn lần.

Bài 14. Cho (a1 , a2 , ..., an ) và (b1 , b2 , ..., bn ) là hai hoán vị của tập n số thực thỏa và
thỏa điều kiện a1 < a2 < ... < an và a1 + b1 < a2 + b2 < ... < an + bn . Chứng minh rằng
ai = bi , ∀i = 1, 2..., n.

Bài 15. Cho 2n số thực phân biệt x 1 , y1 , x 2 , y2 , ..., x n , yn thỏa x k > yk , ∀k = 1, 2, ..., n.
Chứng minh rằng nếu a1 < a2 < ... < an và b1 < b2 < ... < bn là hai hoán vị của tăng
của {x 1 , x 2 , ..., x n } và { y1 , y2 , ..., yn } thì ak > bk , ∀k.

Bài 16. Cho hoán vị a1 , a2 , ..., a2n của {1, 2, ..., 2n} thỏa a1 < a3 < ... < a2n−1 và
a2 < a4 < ... < a2n . Tìm tập tất cả các giá trị của S = |a1 −a2 |+|a3 −a4 |+...+|a2n−1 −a2n |.

Bài 17. Tìm tất cả các giá trị của n sao cho từ tập {1, 2, ..., n, n + 1} người ta có thể chọn
ra n số thỏa a1 , a2 , ..., an sao cho |a1 − a2 |, |a2 − a3 |, ..., |an − a1 | là n số phân biệt.

Bài 18. Cho n ≥ 2. Gọi a1 , a2 , ..., an , a1 là 1 hoán vị vòng quanh của 1, 2, ..., n.

a) Tìm giá trị nhỏ nhất của S = |a1 − a2 | + |a2 − a3 | + ... + |an − a1 | và tính số hoán vị
để đạt giá trị nhỏ nhất.

b) Tìm giá trị lớn nhất của S.

Bài 19. Cho (a1 , a2 , ..., an ) là hoán vị của {1, 2, ..., n}. Tìm giá trị lớn nhất của:

a) S1 = |a1 − 1| + |a2 − 2| + ... + |an − n|

b) S2 = |a1 − a2 | + |a2 − a3 | + ... + |an−1 − an |

Bài 20. Có bao nhiêu hoán vị (a1 , a2 , ..., an ) của {1, 2, ..., n} sao cho tồn tại duy nhất k
thỏa ak > ak+1 .

Bài 21. Tìm số hoán vị của {1, 2, ..., 2014} thỏa i +ai ≤ j +a j với mọi 1 ≤ i < j ≤ 2014.

Bài 22. Cho a1 , a2 , ..., an là dãy các số nguyên không âm sao cho với 1 ≤ i, j và i + j ≤ n
thì
ai + a j ≤ ai+ j ≤ ai + a j + 1

a) Chỉ ra một dãy số thỏa điều kiện trên.

b) Chứng minh rằng tồn tại số thực x sao cho ak = [kx], ∀1 ≤ k ≤ n

Tập san Toán học STAR EDUCATION


66

Bài 23. Cho a1 , a2 , ..., an là một cấp số nhân và b1 , b2 , ..., bn là cấp số cộng. Giả sử
a1 = b1 , an = bn . Chứng minh rằng

a1 + a2 + ... + an ≥ b1 + b2 + ... + bn

Bài 24. Cho 0 ≤ x 0 ≤ x 1 ≤ x 2 ≤ ... ≤ x n ≤ ... là 1 dãy không giảm các số nguyên không
âm. Với mỗi số tự nhiên k, đặt yk là số các số hạng của dãy không lớn k. Chứng minh
rằng với mọi số nguyên dương m, n ta có
n
X m
X
xi + yk ≥ (n + 1)(m + 1)
i=0 k=0

Bài 25. Cho n là một số nguyên dương và a11 , a12 , . . . , ann là các số thực dương thỏa
n
mãn ai j · a ji = 1 với mọi i, j ∈ {1, 2, . . . , n}. Đặt ci = aki với i = 1, 2, . . . , n. Chứng
P
k=1
minh rằng
n
X 1
≤ 1.
c
i=1 i

2. Bài tập số học


Bài 26. (1)Tìm tất cả các số nguyên a, b, c sao cho tồn tại số nguyên dương n thỏa
p n
a + bi 3 p

= c + i 3.
2

Bài 27. (1) Tìm tất cả các số dương n có ít nhất 4 ước số dương thỏa

n | 2φ(n) + 3φ(n) + · · · + nφ(n)

Bài 28. (1)Cho số nguyên dương n. Chứng minh rằng

1. Tồn tại vô hạn các bộ ba số nguyên dương phân biệt (a, b, c) sao cho min(a, b, c) ≥
n và abc + 1 chia hết một trong các số (a − b)2 , (b − c)2 , (c − a)2 .

2. Không tồn tại bộ ba số nguyên dương a, b, c phân biệt sao cho abc + 1 chia hết
nhiều hơn 1 số trong các số (a − b)2 , (b − c)2 , (c − a)2 .

Bài 29. Cho số tự nhiên n > 4.

a) Chứng minh rằng nếu n là hợp số thì không tồn tại hoán vị (a1 , a2 , ..., an ) của tập
{1, 2, ..., n} sao cho a1 , a1 a2 , ..., a1 a2 ...an tạo thành một hệ thặng dư đầy đủ theo
modul n.

b) Tìm số nguyên tố n nhỏ nhất sao cho dãy trên là một hệ thặng dư đầy đủ modul n.

Bài 30. Chứng minh rằng với mọi số tự nhiên n ≥ 2 tồn tại n số nguyên 1 < a1 <
a2 < ... < an sao cho với (b1 , b2 , ..., bn ) là một hoán vị bất kì của {a1 , a2 , ..., an } thì
1 + b2 b3 ...nn chia hết cho b1 .

Tập san Toán học STAR EDUCATION


67

Bài 31. Chứng minh rằng rằng tồn tại dãy vô hạn các số nguyên dương a1 , a2 , ..., an , ...
sao cho a1 + a2 + ... + an là một số chính phương với mọi n.

Bài 32. Chứng minh rằng với mọi n > 1 thì tồn tại hoán vị (a1 , a2 , ..., an ) của {1, 2, ..., n}
sao cho a1 + a2 + ... + ak chia hết cho ak+1 với mọi k = 1, 2, ..., n − 1.

Bài 33. (IMO SL 2008) Cho số tự nhiên n > 1. Tính số hoán vị (a1 , a2 , ..., an ) của tập
{1, 2, ..., n} sao cho 2(a1 + a2 + ...ak ) chia hết cho k với k = 1, 2, ..., n.

Bài 34. Cho số tự nhiên n. Hoán vị (a1 , a2 , ..., an ) của {1, 2, ..., n} được gọi là có tính
chất Pk nếu với mọi i = 1, 2, ..., n−1 thì ai ai+1 +1 là lũy thừa bậc k của một số tự nhiên.

a) Chứng minh rằng tồn tại vô hạn các số tự nhiên n để tồn tại ít nhất một hoán vị có
tính chất P2 .

b) Có tồn tại hay không n để có 1 hoán vị P3 .

Bài 35. Cho số nguyên dương n > 2. Tìm số nguyên dương m nhỏ nhất sao cho tồn tại
dãy a1 , a2 , ..., an các số nguyên dương thỏa hai điều kiện sau:

i) a1 < a2 < ... < an = m.

a12 + a22 2
an−1 + an2
ii) n − 1 số , ..., là các số chính phương.
2 2
Bài 36. Có tồn tại hay không một dãy các số nguyên dương a1 , a2 , ..., an , ... sao cho tổng
của n số liên tiếp chia hết cho n2 .

Bài 37. Cho N là một số nguyên lớn hơn 1. Gọi x là số nguyên dương bé nhất thỏa mãn
tính chất sau: tồn tại một số nguyên dương y, nhỏ hơn x − 1 sao cho x chia hết N + y.
Chứng minh rằng x gấp đôi một số nguyên tố hoặc là lũy thừa của một số nguyên tố.
Chú ý: một số nguyên dương x được gọi là một lũy thừa của số nguyên tố nếu tồn tại
một số nguyên tố p và một số nguyên dương n sao cho x = p n .

Bài 38 (PTNK 2015). Cho A là tập hợp tất cả các số nguyên dương không vượt quá
2016 và nguyên tố cùng nhau với 2016. Có bao nhiêu phần tử a ∈ A để với mỗi phần tử
như vậy, tồn tại số nguyên b sao cho a + 2016b là số chính phương ?

Bài 39 (ĐHSP HN 2015). Ký hiệu S là tập hợp tất cả các số nguyên dương m chỉ có ước
nguyên tố p thoả mãn p ≡ 1 (mod 4). Một số nguyên dương n được gọi là tốt nếu như
với mỗi số m ∈ S, tồn tại các số nguyên dương x, y thoả mãn (x, m) = ( y, m) = 1 và
m | x n + 65 y n + 1. Chứng minh rằng tồn tại vô hạn số tốt. Xác định số tốt bé nhất ?

Bài 40 (IMO 2016). Đặt P(n) = n2 + n + 1. Với mỗi bộ số nguyên dương (a, b), tập hợp
{P(a), P(a + 1), · · · , P(a + b)} được gọi là thơm nếu như không có phần tử nào nguyên
tố cùng nhau với tích của các phần tử còn lại. Tìm số phần tử nhỏ nhất của một tập
thơm.

Bài 41 (APMO 2009). Chứng minh rằng với mọi số nguyên dương k, tồn tại một cấp
a1 a2 an
số cộng gồm các số hạng hữu tỉ là , · · · , , trong đó (ai , bi ) = 1 (1 ≤ i ≤ n) và
b1 b2 bn
a1 , a2 , · · · , an , b1 , b2 , · · · , bn là 2n số nguyên dương đôi một phân biệt.

Tập san Toán học STAR EDUCATION


68

Bài 42 (USAMO 2008). Chứng minh rằng với mỗi số nguyên dương n, tồn tại các số
nguyên k0 , k1 , · · · , kn > 1 đôi một nguyên tố cùng nhau để k0 k1 · · · kn − 1 là tích của hai
số nguyên dương liên tiếp.

Bài 43. Tồn tại hay không 101 số nguyên dương lẻ liên tiếp để mỗi số đều có một ước
nguyên tố lẻ không vượt quá 43 ?

Bài 44 (Russia 2008). Một số nguyên dương được gọi là luỹ thừa đúng nếu số đó có
dạng a m với a, m > 1 là các số nguyên dương. Tìm tất cả các số nguyên dương n > 1 sao
cho tồn tại các số nguyên dương b1 , b2 , · · · , bk không đồng thời bằng nhau để với mọi số
nguyên dương k, số (b1 + k)(b2 + k) · · · (bn + k) là luỹ thừa đúng.

Bài 45 (Bulgaria 2003). Một tập hợp C gồm hữu hạn các số nguyên dương được gọi
là tốt nếu như với số nguyên k bất kỳ, tồn tại hai phần tử a, b ∈ C phân biệt thoả mãn
(a + k, b + k) > 1. Chứng minh rằng nếu tổng các phần tử trong C là 2003, ta có thể
bỏ đi một phần tử khỏi C mà tập hợp mới thu được cũng là tập hợp tốt.

Bài 46 (USA TST 2010). Cho dãy số (an ) (n ≥ 1) được xác định bởi a1 = 1 và:

an = ab n c + ab n c + · · · + ab n c + 1 ∀n ∈ Z+ , n ≥ 2
2 3 n

Chứng minh rằng tồn tại vô hạn số nguyên dương n để 22010 | an − n.

Bài 47 (USA TSTST 2012). Cho f : Z+ → Z+ là một hàm số thoả mãn:

1. Nếu (m, n) = 1 thì ( f (m), f (n)) = 1.

2. Với mọi số nguyên dương n, ta có n ≤ f (n) ≤ n + 2012.

Chứng minh rằng với mọi số nguyên dương n, nếu p | f (n) thì p | n.

Bài 48 (China TST 2014). Cho f : Z+ → Z+ là một hàm số thoả mãn:

1. gcd( f (m), f (n)) ≤ gcd(m, n)2014 .

2. Với mọi số nguyên dương n, ta có n ≤ f (n) ≤ n + 2014.

Chứng minh rằng tồn tại số nguyên dương N để f (n) = n với mọi n ≥ N .

Bài 49 (USA TST 2014). Chứng minh rằng với mọi số nguyên dương n, tồn tại một tập
hợp S gồm n số nguyên dương thoả mãn: với hai phần tử a, b ∈ S phân biệt, ta luôn
có a − b | a và a − b | b nhưng a − b không là ước của bất kỳ phần tử nào khác trong S .

Bài 50 (IMO Shortlist 2014). Cho a1 < a2 < · · · < an là các số nguyên dương đôi một
nguyên tố cùng nhau, hơn nữa a1 ≥ n + 2 là số nguyên tố. Trên đoạn I = [0, a1 a2 · · · an ]
thuộc đường thẳng thực, đánh dấu tất cả các số nguyên là bội của ít nhất một trong các
số a1 , a2 , · · · , an . Các điểm đó phân hoạch I thành các đoạn thẳng nhỏ đôi một rời nhau.
Chứng minh rằng tổng bình phương độ dài của tất cả các đoạn thẳng nhỏ ấy là bội của
a1 .

Tập san Toán học STAR EDUCATION


69

3. Bài tập tổ hợp


Bài 51. Một mảnh vườn hình vuông được chia thành 50×50 ô. Trong mỗi ô vuông 1×1
người ta trồng các loại cây táo, cam, đào. Biết rằng có ít nhất một cây táo gần mỗi cây
cam, có ít nhất một cây táo và một cây cam gần mỗi cây đào, có ít nhất một cây táo một
cây cam, một cây đào gần một ô trống (hai ô gần nhau là hai ô chung cạnh). Chứng
minh rằng số ô còn trống không quá 1000.

Bài 52. (1) Ba tập hợp con phân biệt Si , S j , Sk của một tập có n phần tử được gọi là một
"tam giác". Ta định nghĩa chu vi của "tam giác" là
 
Si ∩ S j ∪ S j ∩ Sk ∪ (Sk ∩ Si ) .
1

Chứng minh số tam giác có chu vi n là 3 2n−1 − 1 (2n − 1).

Bài 53. (1) Cho các số p1 , p2 , p3 là các số nguyên tố phân biệt. Với n nguyên dương, tìm
số ánh xạ f : {1, 2, · · · , 2n} → {p1 , p2 , p3 } sao cho f (1) f (2) · · · f (2n) là một số chính
phương.

Bài 54. Một bảng vuông m × n ô vuông. Trong mỗi ô vuông người ta một số nguyên
không âm. Ta thực hiện phép biển đổi sau: Chọn hai ô chung cạnh và cộng vào số ở mỗi
ô một số nguyên k. Tìm điều kiện cần và đủ để sau một số lần thực hiện ta sẽ được toàn
số 0.

Bài 55. Một bảng vuông m × n được tô bởi hai màu đen và xanh, trong đó 4 ô góc thì 3
ô được tô mà xanh và 1 ô được tô màu đen. Chứng minh rằng tồn tại hình vuông 2 × 2
mà có số ô được tô màu xanh là một số lẻ.

Bài 56. Có n bóng đèn được xếp thành một vòng tròn, có hai trạng thái là bật và tắt.
Trong mỗi lần thay đổi thì chọn một bóng đèn và đổi trạng thái hai bóng đèn bên cạnh,
hỏi với điều kiện các bóng đèn ban đầu như thế nào thì sau một số lần biến đổi chỉ còn
lại một bóng đèn sáng.

Bài 57. Các số từ 1 đến 2015 được ghi thành một hàng theo một thứ tự nào đó, nếu số
đầu tiên là k thì ta sẽ viết k số đầu theo thứ tự ngược lại. Có thể với cách thực hiện đó
thì sau một số lần, số 1 đứng ở vị trí đầu được không? Tại sao?

Bài 58. Một bảng vuông m × n, được tô màu xanh và đen, lúc đầu các số được tô màu
xanh và chỉ có 1 ô góc tô màu đen. Ta thực hiện biến đổi sau, ta chọn một ô màu đen,
đánh dấu ô này và đổi màu các ô chung cạnh. Hỏi sau một số lần thực hiện, ta có thể
đánh chéo tất cả các ô trong bảng vuông được không? Tại sao?

Bài 59. Có 10 ngồi trên một bàn tròn trong đó có một cô gái và 9 chàng trai, có một số
viên kẹo được phân phối cho họ một cách ngẫu nhiên.Các viên kẹo được chuyền qua lại
theo quy tắt sau:

i) Mỗi người chỉ chuyển kẹo qua cho người bên cạnh.

ii) Mỗi lần chuyển kẹo thì người đó phải ăn một viên kẹo.

Tìm số viên kẹo nhỏ nhất sao cho với bất kì cách phân phối kẹo ban đầu nào thì cũng có
cách chuyển ít nhất một viên kẹo đến cho bạn gái.

Tập san Toán học STAR EDUCATION


70

Bài 60. Một bảng vuông 2n × 2n ô, trong đó một nửa được tô màu đen và một nửa được
tô màu đỏ. Trong một phép biến đổi, ta chọn hình vuông 2 × 2 và lấy đối xứng 4 ô đó
theo chiều ngang hoặc theo chiều dọc. Với những giá trị nào của n thì từ một cách tô bất
kì lúc đầu ta có thể đưa về dạng cách tô của bàn cờ.(Đen trắng xen kẻ)
Bài 61. Cho bảng vuông a × b, với a, b là các số nguyên dương. Lúc đầu ta tô màu các ô
vuông bằng các màu đen và trắng xen kẻ như hình bàn cờ. Ta thực hiện việc tô màu cho
các ô vuông theo cách sau: Ta chọn hai ô chung cạnh, nếu ô màu trắng thì tô lại màu
đen, màu đen thì thành màu xanh và màu xanh thì thành màu trắng. Tìm tất cả các
giá trị của a, b để sau khi thực hiện một vài lần thì từ bảng lúc đầu thi được một bảng
ngược lại, tức là ô màu đen thành màu trắng và ô màu trắng thành màu đen.
Bài 62. Cho một bảng vuông n × n, trên đó người ta ghi các số 1 trên các ô ở đường chéo
và ghi số 0 các ô còn lại. Trong mội phép biến đổi ta đổi số các ô cùng dòng hoặc cùng
cột sao cho các ô số 0 thì đổi thành số 1, các ô số 1 thì thành số 0. Chứng minh rằng sau
một số phép biến đổi thì số các số 1 luôn nhiều hơn hoặc bằng n.
Bài 63. Có n bi trắng và n bi đen sắp thành một vòng tròn, chọn một viên bi trắng và từ
viên bi này đánh số các viên bi trắng theo chiều kim đồng hồ từ 1 đến n, chọn một viên
bi đen từ viên bi này đánh số các viên bi đen theo chiều ngược kim đồng hồ từ 1 đến n.
Chứng minh rằng tồn tại n viên bi liên tiếp được đánh số từ 1 đến n (không nhất thiết
phải theo thứ tự)
Bài 64. Cho tập X có n phần tử. Gọi A1 , A2 , ..., Ak là họ các tập con của X thỏa:
i) Mỗi tập Ai có 3 phần tử.

ii) Giao của hai tập bất kì không quá 1 phần tử.
p
Chứng minh rằng tồn tại một tập con có [ 2n] phần tử của X không chứa bất kì tập
nào của họ.
Bài 65. Cho bảng vuông (2n − 1) × (2n − 1) ô vuông. Trên đó người ta ghi các số 1 hoặc
- 1 sao cho với mỗi ô số ghi trong ô đó bằng tích các ô chung cạnh. Có tồn tại hay không
cách ghi mà có ít nhất một số - 1?
Bài 66. Xét số nguyên n > 1. Người ta muốn tô màu tất cả các số tự nhiên bởi hai màu
xanh, đỏ sao cho các điều kiện sau đồng thời thỏa mãn:
i) Mỗi số được tô một màu và mỗi màu được dùng để tô vô số số.

ii) Tổng của n số đôi một khác nhau cùng màu là số có cùng màu đó.
Hỏi có thể thực hiện được phép tô màu nói trên hay không nếu:
a) n = 2014

b) n = 2015
Bài 67. Xét một n−giác đều cùng với tâm của nó. Hai người chơi trò chơi như sau: họ
lần lượt chọn một đỉnh của đa giác rồi nối với một trong hai đỉnh kề hoặc nối với tâm
của hình đa giác đó bởi một đoạn thẳng. Người thắng cuộc là người chơi mà sau lượt
chơi của anh ta thì ở bất kỳ đỉnh nào của đa giác đều có thể di chuyển đến mọi đỉnh còn
lại bằng các đoạn thẳng đã nối ở trên. Với mỗi n ≥ 3, hãy xác định xem ai là người có
chiến lược thắng cuộc.

Tập san Toán học STAR EDUCATION


71

Bài 68. Xung quanh một bàn tròn có n vị trí để ngồi uống trà và mỗi vị trí có một chiếc
bánh nhỏ đặt trên một cái đĩa. Alice tới đầu tiên, ngồi xuống chiếc bàn và ăn chiếc bánh
trước mặt cô ấy. Tiếp đó Mad Hatter tới, anh ta nói với Alice rằng Alice sẽ có một tiệc
trà cô đơn và phải liên liên tục thay đổi chỗ ngồi của mình, trong mỗi lần đổi chỗ cô ấy
phải ăn cái bánh trước mặt mình (nếu nó chưa được ăn). Mad Hatter là một người rất
hách dịch và anh ta nói với Alice rằng khi cô ấy di chuyển lần thứ i với i = 1, 2, ·, n − 1
thì cô ấy phải di chuyển ai vị trí theo danh sách a1 , a2 , · · · , an−1 do anh ta đưa cho cô.
Alice không thích những chiếc bánh và cô ấy tự do trong việc chọn di chuyển cùng chiều
hay ngược chiều kim đồng hồ trong mỗi lần di chuyển của mình. Với những giá trị nào
của n thì Mad Hatter có thể thành công buộc Alice ăn hết những chiếc bánh?

Bài 69. Hai người chơi thay phiên viết mỗi lần một chữ số, từ trái sang phải. Một người
được coi là thua nếu sau lượt của người đó, có một dãy các chữ số

a1 , a2 , . . . , a n

sao cho tồn tại một số nguyên k để ak ak+1 . . . an là bội của 11.
Hỏi người nào có thể thắng và cách chơi của anh ấy như thế nào?

Bài 70. Trên đường tròn cho 2018 điểm. Mỗi điểm được gán một số nguyên sao cho mỗi
số lớn hơn tổng của hai số liền trước nó, theo chiều kim đồng hồ. Xác định giá trị lớn
nhất của số các số dương trong 2018 số đó.

Bài 71. Cho một tập gồm n đường thẳng trên mặt phẳng sao cho không có 2 đường
thẳng trong số chúng song song, hoặc 3 đường thẳng nào trong số chúng đồng quy. Một
điểm gọi là giao điểm nếu điểm này nằm trên hai trong số n đường thẳng trên.

1. Chứng minh rằng tồn tại một đường thẳng trong số n đường thẳng trên sao cho
” (n − 1)(n − 2) —
mỗi nửa phẳng xác định bởi đường thẳng này chứa ít nhất giao
10
điểm.

2. Tìm tất cả các giá trị của n để sao cho số giao điểm trên một trong hai nửa phẳng
” (n − 1)(n − 2) —
tạo bởi đường thẳng tìm được ở trên là gần với giá trị nhất.
10
Bài 72. (APMO 1997, Bài toán 5) n mọi người đang ngồi trong một vòng kết nối. Tổng
cộng nk xu đã được phân phối giữa chúng, nhưng không nhất thiết phải bằng nhau. Di
chuyển là việc chuyển một đồng xu giữa hai người liền kề. Tìm một thuật toán để thực
hiện số lần di chuyển tối thiểu có thể dẫn đến kết quả là tất cả mọi người đều nhận được
cùng một số đồng xu.

Bài 73. Gọi n là một số nguyên dương. Tại mỗi điểm trong số 2n xung quanh một vòng
tròn, chúng ta đặt một cái đĩa với một mặt trắng và một mặt đen. Chúng ta có thể thực
hiện động tác sau: chọn một đĩa đen và lật qua hai đĩa kề với nó. Tìm tất cả các cấu hình
ban đầu mà từ đó có thể có một số bước thay đổi như vậy dẫn đến tất cả các đĩa trừ một
đĩa đều có màu trắng.

Bài 74. Cho một số hữu hạn các thanh dọc có chiều dài 2017 được cố định trên một vài
tấm kim loại. Trên mỗi thanh có một hạt mà nó trượt tự do trên thanh đó. Một vài cặp
hạt được nối với nhau bằng dây đàn hồi. Ant (em) tự do di chuyển trên tất cả các dây.
Còn Ant (anh) tự do di chuyển trên tất cả các dây mà độ chênh lệch chiều cao của các

Tập san Toán học STAR EDUCATION


72

điểm cuối là ±1. Giả sử rằng xuất phát ở bất kì hạt nào, Ant (em) có thể đến bất kì hạt
khác. Cấu hình của hạt mà khi mỗi hạt được đặt ở vị trí là số nguyên và độ cao hai điểm
cuối của bất kì dây nào đều khác nhau, được gọi là cấu hình chấp nhận được. Biết rằng
có ít nhất một cấu hình chấp nhận được của thiết bị này, chứng minh rằng tồn tại một
cấu hình chấp nhận được mà Ant (anh) xuất phát ở bất kì hạt nào cũng có thể đến bất
kì hạt khác.
Bài 75. Gọi p là một hoán vị của {1, 2, · · · , n} sao cho tập {p(k) − k|k = 1, 2, · · · , n} có
đúng hai phần tử. Chứng minh rằng số hoán vị thỏa mãn tính chất trên là σ(n) − τ(n),
trong đó σ(n) là tổng các ước dương của n và τ(n) là số các ước dương của n.

4. Bài tập hình học


Bài 76. (1)Cho tam giác ABC, gọi M , N , P lần lượt là trung điểm của các cạnh BC, AC, AB.
Đặt X , Y, Z là trung điểm các đường cao từ A, B, C. Chứng minh rằng tâm đẳng phương
của các đường tròn ngoại tiếp các tam giác AM X , BN Y, C P Z là tâm đường tròn Euler
của tam giác ABC.
Bài 77. (1) Cho tứ giác ABC D nội tiếp đường tròn tâm O. P là một điểm thuộc miền
trong tứ giác ABC D sao cho tam giác ABP và C DP đồng dạng. Chứng minh OP⊥P E.
Bài 78. Cho tam giác ABC nhọn nội tiếp đường tròn (O). Gọi A1 , B1 , C1 tương ứng chân
đường cao từ A, B, C. Gọi A2 , B2 , C2 là giao điểm của AO, BO, CO với BC, AC, AB. Đường
tròn w a qua A1 , A2 và tiếp xúc với (O) tại Ta . Các điểm Tb , Tc được xác định tương tự.
Chứng minh rằng ATa , BTb , C Tc đồng quy.
Bài 79. Cho tam giác ABC nội tiếp đường tròn w và ngoại tiếp đường tròn C. Đường
tròn tiếp xúc với AB, AC và tiếp xúc trong với w tại P. Đường thẳng d song song với BC
/ BC. Chứng minh rằng ∠BAP = ∠CAK.
và tiếp xúc với C tại K, K ∈
Bài 80. Cho tam giác ABC. Gọi I, I a , I b , I c lần lượt là tâm đường tròn nội tiếp và bàng
tiếp các góc A, B, C.
1. Gọi A1 , A2 , A3 là tiếp điểm của I a với BC, AC, AB. Chứng minh rằng AA1 , BA2 , CA3
đồng quy.

2. Gọi A0 , B 0 , C 0 lần lượt là trung điểm các cạnh BC, AC và AB. Chứng minh rằng
I a A0 , I b B 0 và I c C 0 đồng quy (đặt là X )

3. Gọi B2 , B3 là tiếp điểm của (I b ) với C B, AC;C2 , C3 là tiếp điểm của (I c ) với BC; AC.
I c C3 và I b B3 cắt nhau tại K. Chứng minh I K⊥BC và I a K đi qua tâm đường tròn
ngoại tiếp tam giác ABC.

4. Các đường thẳng A2 A3 và B2 B3 cắt nhau tại Co ; các điểm Ao , Bo được xác định
tương tự. Chứng minh rằng AAo , BBo và C Co đồng quy.

5. Gọi P là hình chiếu của I trên BC. Chứng minh rằng BC3 , C B3 và AP đồng quy.
Bài 81. Cho tam giác ABC có nội tiếp đường tròn w và có I là tâm đường tròn nội tiếp.
Đường tròn w a tiếp xúc ngoài với w và AB, AC tại A1 , A2 ; Các điểm B1 , B2 ; C1 , C2 được
xác định tương tự. Các đường thẳng A1 A2 , B1 B2 và C1 C2 cắt nhau tại thành tam giác
X Y Z.

Tập san Toán học STAR EDUCATION


73

1. Chứng minh I là tâm đường tròn ngoại tiếp tam giác X Y Z.

2. Chứng minh đường thẳng euler của tam giác X Y Z đi qua tâm của w.

Bài 82. Cho tam giác ABC nội tiếp đường tròn w. Tiếp tuyến tại A của w cắt BC tại A0 ;
các điểm B 0 , C 0 được xác định tương tự. Chứng minh rằng A0 , B 0 , C 0 thẳng hàng.

Bài 83. Cho tứ giác ABC D nội tiếp. E là giao điểm của AB và C D; F là giao điểm của
BC và AD.

1. Chứng minh rằng đường tròn ngoại tiếp các tam giác EBC, EAD, FAB, F C D cùng
đi qua một điểm P.

2. Tiếp tuyến tại P của (FAB) cắt AB tại X ; Tiếp tuyến tại P của (EBC) cắt BC tại
Y , tiếp tuyến tại P của (FAD) cắt AD tại Z và tiếp tuyến tại P của (F C D) cắt C D
tại T . Chứng minh X , Y, Z, T thẳng hàng.

3. Chứng minh rằng đường tròn ngoại tiếp hai tam giác (PY Z) và (P X T ) tiếp xúc
nhau.

Bài 84. Cho tứ giác ABC D khác tứ giác nội tiếp. Các điểm X , Y, Z, T thuộc các đường
thẳng AB, BC, C D và DA thỏa

AX DA2 BY AB 2 C Z BC 2 DT C D2
=− , = − , = − , = −
XB BC 2 Y C C D2 Z D DA2 TA AB 2

Chứng minh rằng X , Y, Z, T cùng thuộc một đường tròn.

Bài 85. Gọi H là trực tâm của tam giác ABC. Gọi d1 , d2 là hai đường thẳng qua H
vuông góc nhau. d1 cắt BC và đường thẳng AB kéo dài tại D, Z; đường thẳng d2 cắt BC
và đường thẳng AC kéo dài tại E, X . Lấy điểm Y sao cho Y D||AC, Y E||AB. Chứng minh
rằng X , Y, Z thẳng hàng.

Bài 86. Cho tam giác ABC. Gọi A1 , B1 , C1 lần lượt là chân đường cao từ A, B, C. Gọi
A0 , B 0 , C 0 là trung điểm AA1 , BB1 , C C1 . Đường thẳng B 0 C 0 cắt AB, AC tại A b , Ac . Gọi Oa
là tâm đường tròn ngoại tiếp tam giác AA b Ac ; các điểm Ob , Oc được xác định tương tự.
Chứng minh rằng Oa , Ob , Oc thuộc đường tròn ngoại tiếp tam giác A0 B 0 C 0 .

Bài 87. Cho tam giác ABC có đường tròn nội tiếp w tâm I.Đường tròn w a tiếp xúc ngoài
với w và AB, AC tại A1 , A2 ; Các điểm B1 , B2 ; C1 , C2 được xác định tương tự. Các đường
thẳng A1 A2 , B1 B2 và C1 C2 cắt nhau tại thành tam giác X Y Z. Chứng minh rằng tâm nội
tiếp của X Y Z, tâm ngoại tiếp của X Y Z và I thẳng hàng.

Bài 88. Cho tam giác ABC cân tại A, M là trung điểm BC. D là điểm thuộc đường cao
từ C nằm ngoài tam giác và BD = BM . Đường trung trực BD cắt đường tròn ngoại tiếp
tam giác ABM tại K sao cho B, K khác phía đối với AM . Tính ∠DM K.

Bài 89. Cho tam giác ABC nội tiếp đường tròn (O) trực tâm H. Gọi H a , H b , H c là
chân các đường cao từ A, B, C. Gọi P là điểm thuộc đường thẳng OH. Các đường
thẳng AP, BP, C P cắt (O) tại A1 , B1 , C1 . Gọi A2 , B2 , C2 là điểm đối xứng của A1 , B1 , C1
qua H a , H b , H c . Chứng minh rằng H, A2 , B2 , C2 đồng viên.

Tập san Toán học STAR EDUCATION


74

Bài 90. Cho tam giác ABC, gọi A1 , B1 , C1 lần lượt là chân đường cao hạ từ A, B, C. Gọi
A2 , B2 , C2 là chân các đường phân giá trong của tam giác A1 B1 C1 . Gọi da là đường thẳng
qua A vuông góc với B2 C2 ; các đường thằng d b , dc được xác định tương tự. Chứng minh
da , d b , d2 đồng quy.

Bài 91. Cho tam giác ABC nội tiếp đường tròn tâm O. Gọi Oa , Ob , Oc lần lượt là tâm
đường tròn ngoại tiếp của các tam giác OBC, OAC và OAB.

1. Chứng minh rằng AOa , BOb , CO c đồng quy tại K (điểm K gọi là Kosnita).

2. Chứng minh K là điểm đẳng giác của tâm đường tròn Euler.

Bài 92. Cho tam giác ABC có H, O, N lần lượt là trực tâm, tâm ngoại tiếp và tâm đường
tròn euler, K là điểm konista. Gọi Nb = BN ∩ AC; Nc = C N ∩ AB và A1 = Nb Nc ∩ BC. Gọi
Ka là điểm đối xứng của K qua BC.

1. Chứng minh H Ka ⊥AA1 .

2. Định nghĩa K b , B1 tương tự Ka , A1 . Đường thẳng qua O vuông góc với AA1 , BB1 cắt
N Ka , N K b tại X , Y . Chứng minh X Y ||Ka K b .

Bài 93. (2003 USA TST P6) Cho tam giác ABC nhọn, khác tam giác cân. Gọi H1 , H2 , H3
lần lượt là chân đường cao từ A, B, C. Đường tròn nội tiếp tam giác tiếp xúc với các
cạnh BC, AC, AB tại T1 , T2 , T3 . Gọi P1 là điểm thuộc đường thẳng H1 H2 sao cho tam giác
H1T1 P1 cân tại H1 ; các điểm P2 , P3 được xác định tương tự. Chứng minh đường tròn
ngoại tiếp các tam giác T1 P1 T2 , T2 P2 T3 , T3 P3 T1 cùng đi qua một điểm chung.

Bài 94. Cho tứ giác ABC D,E = AD ∩ BC, F = AB ∩ C D, Gọi X = AC ∩ BD, Y = BD ∩


E F, Z = E F ∩ AC .Gọi M là điểm Migel của tứ giác toàn phần ABC DE F , chứng minh M
nằm trên đường tròn Euler của tam giác X Y Z.

Bài 95. Cho tam giác ABC các đường cao AD, BE, C F . Gọi A1 , B1 , C1 là trung điểm các
đường cao AD, BE, C E. B1 C1 cắt AB, AC tại C2 , B2 ; gọi Oa là tâm đường tròn ngoại tiếp
tam giác AB2 C2 ; các điểm Ob , Oc được xác định tương tự. Chứng minh rằng 6 điểm
A1 , B1 , C1 , Oa , Ob , Oc cùng thuộc một đường tròn.

Bài 96. (Sharygin2015-P4) Cho tam giác ABC đường cao AD, BE, C F , gọi X , Y, Z là
tiếp điểm của đường tròn bàng tiếp đỉnh A, B, C với BC, CA, AB. Giả sử E F tiếp xúc với
đường tròn nội tiếp tam giác ABC. Chứng minh rằng D nằm trên đường tròn ngoại tiếp
X Y Z.

Bài 97. Cho tam giác ABC nội tiếp đường tròn tâm O, D, E thuộc cạnh AB, AC. Gọi
M , N , P lần lượt là trung điểm BE, C D và DE, K là hình chiếu của O trên DE. Chứng
minh 4 điểm M , N , P, Q đồng viên.

Bài 98. (China TST 2021). Cho đường tròn w và dây cung U V , tiếp tuyến tại U, V cắt
nhau tại A, gọi O là tâm của (AU V ) và D là trung điểm AU. Đường tròn w0 tâm C qua
A, U cắt cung U V tại P và cắt AV tại Q sao cho trung điểm R của cung nhỏ PQ thuộc
đường cao từ U của tam giác AU V . PQ cắt AU tại L, N là trung điểm AL, M là trung
điểm DR, X là hình chiếu của N trên OM .
Chứng minh tứ giác DM T C nội tiếp.

Tập san Toán học STAR EDUCATION


75

Bài 99. (VN TST 2013) Cho tứ giác ABCD có các cạnh đối không song song nội tiếp
đường tròn (O, R). Gọi E là giao điểm hai đường chéo và đường phân giác góc AEB cắt
các đường thẳng AB, BC, CD, DA lần lượt tại các điểm M, N, P, Q.

1. Chứng minh rằng các đường tròn (AQM ), (QM N ), (C N P), (DPQ) cùng đi qua một
điểm duy nhất. Gọi điểm đó là K.

2R2
2. Đặt min AC, BD = m. Chứng minh rằng OK ≤ p .
4R2 − m2
Bài 100. (VN 2013) Cho tam giác ABC nhọn không cân có A bằng 45◦ . Các đường cao
AD, BE, C F đồng quy tại trực tâm H . Đường thẳng E F cắt đường thẳng BC tại P . Gọi
I là trung điểm của BC ; đường thẳng I F cắt P H tại Q.

1. Chứng minh rằng ∠IQH = ∠AI E.

2. Gọi K là trực tâm của tam giác AE F và (J) là đường tròn ngoại tiếp tam giác K P D
. Đường thẳng C K cắt (J) tại G ,đường thẳng I G cắt (J) tại M , đường thẳng J C
cắt đường tròn đường kính BC tại N . Chứng minh rằng các điểm G, M , N , C cùng
thuộc một đường tròn.

Tập san Toán học STAR EDUCATION


Hình học xạ ảnh
bắt nguồn từ hội họa

Trần Tín Nhiệm (Sưu tầm)

Hầu như tất cả họa sĩ đều có thể vận dụng một cách thành thạo nguyên lý phối cảnh
để biểu diễn hình khối, bởi vì phối cảnh có thể giúp cho người vẽ quan sát chính xác,
khoa học đối với hình thái của vật thể?

Nói từ góc độ hội họa, cái gọi là phối cảnh tức là nhìn vật thể qua một tấm kính phẳng
dựng đứng giữa mắt người và vật thể. Lúc này ta có thể coi tấm kính phẳng là bản vẽ.
Hình thu lại của vật thể nhìn thấy trên tấm kính phẳng chính là hình tượng mà ta cần
trên bản vẽ.

Nhà thơ Đỗ Phủ (712 - 770) đời Đường (618 - 907), quê ở tỉnh Hồ Bắc, khi miêu tả
quang cảnh xung quanh "thảo đường" (nhà tranh), đã để lại một tuyệt cú thiên cổ:

"Lưỡng cá hoàng ly minh thúy liễu,


Nhất hàng bạch lộ thướng thanh thiên.
Song hàm Tây Lĩnh thiên thu tuyết,
Môn bạc Đông Ngô vạn lý thuyền".

Dịch ý là:

"Hai con hoàng ly hót trên cây liễu biếc,


Một hàng cò trắng bay trên trời xanh.
Qua cửa sổ nhìn tuyết nghìn năm của ngọn Tây Côn Lĩnh,
Qua cửa đi nhìn thuyền Đông Ngô đậu dài vạn dặm."

76
77

Thảo đường là nơi ông thường đọc sách.

Trong hội họa, một điểm trên bản vẽ hướng đúng vào mắt người vẽ gọi là tâm điểm
và các đường thẳng vuông góc với bản vẽ đều thu về tâm điểm. Hình 20 - 1a là bản vẽ
mặt bằng (nhìn từ trên xuống). Trên bản vẽ này có ba đường ray tàu hỏa song song,
bên phải ba đường ray là một hàng cây. Hình 20 - 1b là bản vẽ phối cảnh của hình 20
- 1a khi người ta đứng ở điểm "X": hàng cây, đường ray và hàng cột điện vuông góc
với mặt bản vẽ mà duỗi ra phía xa, đều giao nhau và hợp lại ở tâm điểm.

Thời kỳ Văn hóa Phục Hưng ở châu Âu, cùng với thành
tựu của phép phối cảnh, giao hòa ánh sáng, với sự vẻ
vang của lịch sử hội họa, rất nhiều họa sĩ nổi tiếng
xuất hiện, chẳng hạn Leonardo de Vinci (15.4.1452 -
2.5.1519) người Italia, bằng những kỹ xảo và tài năng
phi phàm, đã có được những cống hiến tuyệt vời cho
sự nghiên cứu phép phối cảnh. Thành quả của họ
ảnh hưởng rất nhanh tới hình học và thai nghén một
nhánh hình học mới : hình học xạ ảnh (hình học hình
chiếu).

Cái gọi là hình học xạ ảnh là chỉ chùm


tia sáng tỏa chiếu xuất phát từ trung tâm
O làm cho hình vẽ Ω trên mặt phẳng
Q thu được ảnh cắt Ω0 trên mặt phẳng
P (hình 20-2). Vậy Ω0 gọi là hình chiếu
trên mặt phẳng P của Ω từ trung tâm O.
Hình học xạ ảnh chính là môn học nghiên
cứu tính chất bất biến trong phép chiếu
nói trên. Rõ ràng nó vừa không giống với
hình học Euclid mà học sinh trung học
đang học, cũng không giống với hình học
trên màng cao su đã giới thiệu ở các mục
trước.

Đặt nền móng cho sự ra đời của hình học xạ ảnh


là hai nhà toán học người Pháp : Gérard Desargues (2.3.1591 - 10.1661) và Blaise
Pascal (19.6.1623 - 19.8.1662).

Tập san Toán học STAR EDUCATION


78

Năm 1636, G.Desargues đã công bố cuốn sách : "Phương pháp chung biểu diễn vật thể
bằng phép phối cảnh". Trong cuốn sách này tác giả lần đầu tiên đã đưa ra khái niệm
"thước
đo chiều cao, chiều rộng và chiều sâu", từ đó gắn lý
thuyết hội họa với khoa học nghiệm túc. Điều không thể
tưởng tượng được là đối với tiến bộ khoa học này, khi đó
lại bị sự công kích từ nhiều phía, làm cho G.Desargues
nổi giận. Ông tuyên bố công khai rằng : Hễ ai tìm thấy
sai sót trong phương pháp của ông, nhất loạt ông sẽ
thưởng 100 đồng tiền Tây Ban Nha, ai có thể nêu ra
phương pháp tốt hơn, bản thân ông bằng lòng chi 1000
franc. Đây quả là một loại chế nhạo đối với lịch sử
!

Năm 1639, G.Desargues nghiên cứu thiết diện phẳng


của mặt nón (thiết diện nón, thiết diện conic)
(hình 20 - 3), đã có đột phá mới. Ông trình
bày ba loại đường cong bậc hai đều có thể do
mặt phẳng cắt hình nón mà có, từ đó có thể
coi ba loại đường cong đều là hình vẽ phối cảnh
của đường tròn. Điều này khiến cho những nghiên
cứu về thiết diện nón có một hình thức đặt biệt
gọn.

Có điều là, các tác phẩm nói trên của G.Desargues sau này
lại .không may bị thất truyền, mãi tới hơn hai trăm năm
sau, vào một ngày của năm 1845 , nhà toán học Michel
Chasles (15.11.1793 - 18.12.1880) người Pháp, do một
sự ngẫu nhiên, ở một quán sách cũ tại Paris, đã kinh ngạc phát hiện bản sao gốc của
G.Desargues, từ đó làm cho các thành quả bị mai một của G.Desargues, được toả sáng
trở lại.

G.Desargues sở dĩ được lưu danh lịch sử, còn do định lý sau đây :

"Nếu các cạnh tương ứng của các hình ba đỉnh A, B, C và A 1 , B1 , C1 giao nhau tại ba
điểm P, Q, R nằm trên một đường thẳng thì ba đường thẳng nối ba đỉnh tương ứng đi
qua một điểm S" (hình 20 - 4). Người ta gọi đây là "định lý Desargues thứ nhất".

Tập san Toán học STAR EDUCATION


79

Điều thú vị là : nếu sửa "đỉnh" trong "Định lý Desargues thứ nhất" thành "đường thẳng"
và sửa "đường thẳng" thành "đỉnh" thì định lý thu được vẫn đúng :

"Nếu ba đường thẳng nối ba đỉnh tương ứng của các hình ba đỉnh A, B, C và A 1 , B1 , C1
đi qua một điểm S thì ba cạnh tương ứng của các hình ba đỉnh đó giao nhau tại ba
điểm P, Q, R nằm trên cùng một đường thẳng" (hình 20 - 4).

Định lý Desargues đúng đối với các hình có ba đỉnh nằm trên cùng một mặt phẳng,
cũng như đối với các hình ba đỉnh nằm trên các mặt phẳng khác nhau.

Các hình ba đỉnh nêu trong "Định lý Desargues" được gọi là các hình tam giác, vì vậy
"Định lý Desargues" thường được gọi là "Định lý Desargues về các hình tam giác".

Trong hình học xạ ảnh, hiện tượng nói ở trên có tính phổ biến. Nói chung, chuyển các
từ ngữ trong các mệnh đề đã biết hoặc trong xây dựng bản vẽ theo "từ điển" sau đây :

• Điểm • Đường thẳng

• Ở trên ... • Đi qua ...

• Đường thẳng nối liền hai điểm • Giao điểm của hai đường thẳng

• Điểm chung • Đường thẳng

• Hình bốn đỉnh • Hình bốn cạnh

• Tiếp tuyến • Tiếp điểm

• ... • ...

thì sẽ được một mệnh đề "đối ngẫu" (đối nhau). Hai mệnh đề "đối ngẫu" với nhau thì
hoặc là cùng đúng hoặc là cùng sai. Đây chính là "nguyên lý đối ngẫu" có một không
hai trong hình học xạ ảnh.

Chắc bạn đọc còn nhớ câu chuyện về Sofia Vaxilevna Covalevxkaia (15.1.1850 -
10.2.1891) nhà toán học nữ xuất sắc người Nga đã kể ở Tập Hai "Cái đã biết trong
cái chưa biết" của bộ sách này. Ở đó bạn đọc đã thấy được phần nào về sự cản trở của
người cha trên con đường mà bà tìm đến với toán học. Rồi cũng trong tập sách đó đã
kể câu chuyện về người cha vô tình vì thương con, không muốn con đi theo vết xe
tuyệt vọng của mình, mà cản trở con say mê nghiên cứu toán học. Đó là câu chuyện
về nhà toán học János Bolyai (15.12.1802 - 27.1.1860) người Hungari, đã sáng lập
ra hình học phi Euclid (hình học hypecbolic).

Các câu chuyện như vậy rất nhiều, chứng tỏ con đường đến với khoa học nói chung
và toán học nói riêng cũng lắm trắc trở. Thế nhưng hầu hết các nhà khoa học thời còn
trẻ đều vượt qua được để tiếp tục đi theo con đường mà mình đã say mê. Sau đây là
câu chuyện về nhà toán học "thần đồng" B.Pascal :

Cha của B.Pascal cũng là một nhà toán học nhưng không biết nguyên nhân tại sao
mà lại cực lực phản đối B.Pascal học toán, thậm chí thu hết sách toán học. Nào ngờ,

Tập san Toán học STAR EDUCATION


80

tất cả những cái đó lại càng làm cho B.Pascal say sưa suy nghĩ về những điều thần bí
của toán học. Bởi vậy, lúc còn nhỏ ông đã tự mình chứng minh lại được nhiều định lý
trong hình học phẳng. Chẳng hạn : " Tổng ba góc trong của tam giác bằng 180◦ ".

Thiên tài toán học của B.Pascal đã làm cho người cha cảm động ứa nước mắt và từ
đó ông đã đổi thái độ : chẳng những bỏ cấm đoán, mà còn đưa con đi dự hội thảo do
M.Mersenne - người sáng lập Viện Hàn lâm khoa học Pháp - chủ trì. Khi đó B.Pascal
mới 14 tuổi.

Năm 1639, B.Pascal đã phát hiện định lý sau đây khiến tên ông lưu danh lịch sử :

"Nếu A, B, C, D, E và F là 6 điểm bất kỳ trên đường bao của một thiết diện nón thì ba
giao điểm của AB và D E, B C và E F, CD và FA cùng nằm trên một đường thẳng" (hình
20 - 5 ).

Định lý này là tuyệt diệu, nó tỏ rõ một thiết diện nón chỉ cần năm điểm là có thể xác
định được, điểm thứ sáu có thể tìm được nhờ điều kiện cộng tuyến (cùng nằm trên
một đường thẳng). Suy luận từ định lý này người ta đã tìm được hơn 400 hệ quả,
ngang với một bộ sách lớn.

Nào ngờ, thành quả huy hoàng này lại gây nên sự hoài
nghi của một số người, trong đó có R. Descartes. R.Descartes
không tin rằng đây lại là suy nghĩ của một cậu bé 16 tuổi,
mà cho rằng, đây là công sức của người cha cậu. Có điều
là thành quả của B.Pascal về sau càng nhiều : 19 tuổi phát
minh ra máy tính số học dầu tiên (về sau người ta thường gọi
là máy tính cơ điện), 23 tuổi phát minh ra định luật cường
độ nén của chất lỏng, 31 tuổi (cùng với Pierre de Fermat
(3.8.1601 - 15.1.1665) người Pháp) sáng lập ra lý thuyết xác
suất, 35 tuổi thu được thành quả to lớn trong nghiên cứu
đường hyperbol, .... Với hàng loạt thành tựu liên tục này, ông
đã khuất phục được tất cả những người hoài nghi. Đến lúc này không ai không đồng
thanh ca ngợi con người thiên tài này của đất nước Pháp.

Điều bất hạnh là từ khi hai nhà toán học dẫn đường của môn hình học xạ ảnh lần lượt
qua đời (vào năm 1661 và năm 1662) thì sau đó việc nghiên cứu môn hình học này
không được người ta chú trọng, vì thế nó đã lắng đi khoảng một thế kỷ rưỡi, mãi đến
khi xuất hiện nhà toán học J.V.Poncelet người Pháp.

Tập san Toán học STAR EDUCATION


Giới thiệu các lớp chuyên đề

Song song với quá trình dạy học cho học sinh đại trà, vấn đế bồi dưỡng học sinh có
năng khiếu luôn được Ban giám đốc Star Education đặc biệt quan tâm. Qua nhiều năm
tổ chức, hiện nay trung tâm có nhiều học sinh tham gia học tập tại các lớp chuyên đề
bồi dưỡng học sinh giỏi, nhiều em trong các lớp học đó đã tiếp tục chọn môn chuyên
để học tập ở bậc cao hơn.
Môn toán hiện là đầu tàu vì có nhiều lớp chuyên đề nhất rải đều ở các khối lớp, thu
hút nhiều học sinh tham gia học tập nhất, bài viết này giới thiệu cho bạn đọc hiểu
thêm về các lớp Chuyên đề toán hiện đang được tổ chức tại trung tâm, cũng như các
định hướng tương lai của các lớp này.

• Lớp đội tuyển: Lớp đội tuyển (vòng 1, vòng 2) là lớp học dành cho các em là
thành viên đội tuyển các tỉnh chuẩn bị cho kì thi Học sinh giỏi Quốc gia (vòng
1) và kì thi TST (vòng 2), lớp này góp phần hỗ trợ phần nào cho các em trong
quá trình học tập và ôn luyện ngoài giờ trên lớp chính thức, để các em có thành
tích tốt nhất trong kì thi VMO và TST.

• Lớp 10 Chuyên đề: Lớp 10 chuyên đề dành cho các bạn học sinh đam mê môn
toán, bắt đầu khi các em từ khi các em vào lớp 10 và kết thúc là kì thi chọn đội
tuyển của các trường, tỉnh. Lớp giúp định hướng học tập cho các em trong giai
đoạn đầu THPT, dạy kiến thức và kĩ năng cho các em có năng khiếu, muốn đi
xa với môn toán, không phải là lớp để luyện thi đại học điểm cao, bồi dưỡng
cho các em đam mê môn toán, dạy cho các em có nền tảng toán tốt để sau này
có thể chọn môn toán là môn chủ đạo để nghiên cứu ở bậc cao hơn. Ngoài ra
các em tham gia lớp học có thể tham dự các kì thi HSG của trường, tỉnh, và các
kì thi quốc tế tổ chức tại Việt Nam như IGO, Sharygin, ICO,...

• Lớp 9 Chuyên đề: Đây là lớp có truyền thống nhất tại Star Education, lớp bồi
dưỡng học sinh giỏi lớp 9, cùng với lớp 9TC1, lớp học này dạy kiến thức, kĩ năng
toán học truyền đam mê, tạo tiền đề cho các em học lên cấp ba, tham dự các
kì thi HSG của tỉnh thành phố và bồi dưỡng các em thi vào các lớp chuyên toán
của TPHCM, cũng như của các tỉnh. Lớp học có rất nhiều em đã đậu cao vào lớp
10 chuyên chọn của trường PTNK, LHP, ...

• Lớp 8 Chuyên đề: Lớp học dạy các chuyên đề khó của lớp 8, bổ sung thêm cho
lớp 8TC, giúp các em bắt đầu làm quen với toán chuyên, truyền lửa đam mê học
toán cho các em, tạo điều kiện cho các tham gia các kì thi HSG trong và ngoài
nước.

• Lớp 67 Chuyên đề: Lớp này là lớp học non trẻ nhất của trung tâm, tuy nhiên
đã thu hút khá nhiều học sinh tham gia, lớp học này bước đầu giúp các em làm
quen với tư duy toán, những bài toán logic, suy luận sẽ được ưu tiên hơn là kiến

81
82

thức hay kĩ năng, các bài toán được chọn lọc để các em tập suy luận, tư duy, từ
đó bước đầu hình thành niềm yêu thích đối với môn toán và có thể học tốt các
môn học khác.
Mục tiêu quan trọng của các lớp chuyên đề là phát hiện và bồi dưỡng học sinh, tạo
tiền đề để các em học toán ở bậc cao hơn, chứ không phải là các kì thi HSG, do đó các
lớp học được tổ chức bài bản, với nội dung được chọn lọc cẩn thận, toán là môn khó,
học xong cần phải có thời gian suy ngẫm và tự hoàn thiện, do đó khi tổ chức các lớp
này chúng tôi luôn nhắm tới các mục đích lâu dài chứ không phải là các lớp luyện thi
trong thời gian ngắn.
Trong giai đoạn sắp tới, định hướng của các lớp chuyên đề là sẽ chuyển sang hình
thức online, vì với các em học sinh giỏi, các em có thể tập trung và chủ động học tập
tốt hơn các em khác nên hình thức online vẫn không ảnh hưởng đến chất lượng đào
tạo, ngoài ra một vài giáo viên của trung tâm đang học tập và làm việc nước ngoài
nên hình thức này việc học tập sẽ không bị gián đoạn, và mục tiêu quan trọng nhất
của việc tổ chức online là muốn mở rộng đối tượng học sinh tham gia cho các tỉnh
khác ngoài học sinh TPHCM, các em ở tỉnh đam mê học toán có thể đăng kí tham gia
các lớp chuyên đề của Star Education.
Thành phần giảng dạy các lớp chuyên đề hiện nay khá đông, bên cạnh các giáo viên
có nhiều kinh nghiệm dạy chuyên toán như thầy Lê Phúc Lữ, thầy Nguyễn Tăng Vũ,
thầy Vương Trung Dũng,... nhiều bạn trẻ dù đang là sinh viên nhưng có những thành
tích rất tốt khi còn học phổ thông, kĩ năng SP rất tốt như Tiến Hoàng, Công Thành, ...
• Chuyên toán THPT: Nguyễn Tăng Vũ, Vương Trung Dũng, Nguyễn Ngọc Duy,
Lê Phúc Lữ, Trần Hà Sơn, Nguyễn Tiến Hoàng (ĐHKHTN), Nguyễn Huy Hoàng
(HCB IMO 2015), Phạm Nguyễn Mạnh (HCB IMO 2016), Nguyễn Mạc Nam
Trung (HCB IMO 2019-ĐHKHTN), Nguyễn Công Thành (VinUni).

• Chuyên Toán THCS: Nguyễn Tăng Vũ, Nguyễn Phi Hùng (GV PTNK), Lương
Xuân Vinh, Nguyễn Huy Hoàng, Trần Tín Nhiệm.

Tập san Toán học STAR EDUCATION

You might also like